Internal Medicine Shelf

¡Supera tus tareas y exámenes ahora con Quizwiz!

26 year old man. 2 week history of persistent abdomianl pain in RUQ, low grade fever and anorexia. Recently immigrated from MExico. Several months ago had bloody diarrhea for severeal days, resolved spontaneously. Temp is 101, tenderness to palpation in right upper quadrant. CT of abdomen shows 2x3 cm abscess in the right lobe of the liver. What is this and how to treat?

- Liver abscess most likely from Entomoeba - Developing country, classically causes amoebic colitis (bloody diarrhea) - Tx. metronidazole

Mechanism of hypotension in pancreatitis?

- Local and systemic vascular endothelial injury - Causes vasodilation, increased vascular permeability, and plasma leak into the retroperitoneumm resulting in systemic hypotension

What treatment has been shown to prolong survival and improve quality of life in patients with COPD with signficant chronic hypoxemia?

- Long-term oxygen therapy

First step when somebody has V. tach?

- Look for identifiable cause (ex. electrolytes)

Best initial treatment for pulmonary hypertension from left ventricular dysfunction?

- Loop diuretics + ACEi (or ARB)

Treatment of chemotherapy-induced diarrhea?

- Loperamide

43 year old man comes in with generalized bone pain. History of Crohn disease and underwent extensive small bowel resection several years ago. PE shows bony tenderness of the lower spine, pelvis, and lower extremities. Mild weakness of proximal limb muscles. Radiographs show diffuse osteopenia of teh spine and pseudofractures in both femoral necks. What is this?

- Osteomalacia - Crohn disease with bowel resection can cause chronic deficiency of vitamin D leading to osteomalacia

Serum calcium, serum phosphate, serum alk phos, and serum parathyroid in: - Osteomalacia/rickets - Osteoporosis - Osteopetrosis - Paget disease of the bone

- Osteomalacia/rickets: decreased phosphate, decreased calcium, increased alk phos, increased PTH - Osteoporosis: everything normal - Osteopetrosis: everything essentially normal - Paget's disease: normal phosphate, normal calcium, increased alk phos, normal PTH

Common cause of hearing loss in pregnant patients?

- Otosclerosis

Mechanism of G6PD deficiency injury?

- Oxidative injury

All patients with acute cocaine toxicity and myocardial ischemia should be initially treated with what?

- Oxygen and benzodiazepines

CD4 count most associated wtih increased risk of PCP?

- PCP pneumonia most common in CD4<200

What is defined as high PEEP used for ARDS and what complication can this cause and why?

- PEEP greater than about 8-9 - Causes barotrauma which can lead to pneumothorax

Describe the appearnce of these brain lesions in HIV/AIDS: - PML - Toxoplasmosis - Primary CNS lymphoma

- PML: asymmetric, hypodense, nonenhancing white matter lesions wtih no surrounding edema - Toxoplasmosis: ring-enhancing lesions with edema - Primary CNS lymphoma: well-defined, focal enhancing lesion

Treatment for a duodenal ulcer?

- PPI + Antibiotics (to eradicate H. pylori because that is almost always what causes them)

Explain the mechanism of each type of pneumonia vaccine.

- PPSV23 - polysaccharide capsular vaccine - T-cell independent B cell response - PCV13: polysaccharide-protein conjugate vaccine - T-cell dependent B cell response

Next most important step in evaluating hypercalcemia?

- PTH level - If high-normal: primary hyperparathyroidism, most likely - If suppressed - concern for malignancy

Measurements of CVP (right sided preload), PCWP (left-sided preload), cardiac index (LV output), and SVR (afterload) in hypovolemic, cardiogenic, obstructive, and septic shock.

- Hypovolemic: decreased CVP, decreased PCWP, decreased cardiac index, increased SVR - Cardiogenic: increased CVP, increased PCWP, decreased cardiac index, increased SVR - Obstructive: increased CVP, decreased PCWP, decreased cardiac index, increased SVR - Septic shock: decreased CVP, decreased PCWP, increased cardiac index, decreased SVR

32 year old woman. fever, chills pleuritic chest pain and sOB for 3 days. HIV positive 1 year ago, but doesn't come to appointments or take medications. Last CD4 count was 1 month ago. uses IV heroin and cocaine. Exam shows crackles in both lung fields. Extremities show severeal needle marks. Imaging of the chest receals multiple nodular lesions with small cavities. Waht is likely going on?

- IE causing septic emboli - IV drug user with cavitary lesions

When do you screen earlier for colon cancer?

- IF a first degree relative had colon cancer - screen at 40 or 10 years before the age of diagnosis in that family member (whichever comes first)

Different EKG leads and blood supply.

- II, III, avf: inferior leads - RCA - I, avl , V5/6: lateral leads - left circumflex - V1-V4: V1/2 (septal), V3/4 (anterior) - LAD

Cardioembolic stroke from infective endocarditis from IV drug use. How to treat?

- IV antibiotics - Dont' do surgery unless really large vegetation or not going away with antibitoics - Don't use an anticoagulant as it increases the risk for intracranial hemorrhage

Managmenet of acute pancreatitis.

- IV fluids and supportive care (analgesics)

When is the only time you give tetanus IG with the vaccine for prophylaxis after a wound?

- If the patient is unimmunized, unsure about vaccine status, or didn't receive all 3 vaccines origianlly AND the wound is dirty or severe - You don't even give if it they meet those criteria but the wound is small and clean

Compare Iga nephropathy vs postinfectious glomerulonephritis

- IgA nephropathy: usually within 5 days of URI; more common in men age 20-30; NORMAL serum complements - Postinfectious: usually 10-21 days after URI; more common in children ages 6-10 but can occur in adults; LOW C3 complement

Mechanism for increased aspiration risk in patients with dementia?

- Impaired swallowing and cough reflex

What is the rare disease assumption?

- In a case control study, if the outcome is uncommon in the population, the odds ratio is a close approximation of the relative risk

What is the Stemmer sign?

- Inability to lift the skin on the dorsum of the second toe - Lymphadema sign

What force primarily causes cirrhotic ascites?

- Increased capillary hydrostatic pressure

Why can patients wtih an AV fistula develop high-output heart failure?

- Increased cardiac in reponse to a redcution in systemic vascular resistance (ex. the fistula) - Increased CO and decreased SVR also lead to increased cardiac preload

What is a leukemoid reaction? Vs. CML?

- Leukocyte count >50,000, usually caused by severe infection, predominance of neutrophil precursors (more mature so metamyelocytes > myelocytes), absolute basophilia not present - CML: leukocyte often >100,000, caused by BCR-ABL fusion, more less mature precursors (myelocytes > metamyelocytes), absolute basophilia present

Subacute thryoiditis? (De Quervain)

- Likely postviral inflammatory process - Prominent fever and hyperthyroid symptoms - Painful/tender goiter - Elevated ESR/CRP - Low radioiodine uptake

Complication and time course after each cause of MI: - RCA - LAD or RCA - LAD

- RCA: acute right ventricular failure, acute or within 3-5 days - papillary muscle rupture - LAD (apical septal) or RCA (basal septal): acute of within 3-5 days - interventricular septum rupture - LAD: within 5 days or up to 2 weeks - free wall rupture; up to several months - left ventricular aneurysm

What hyperthyroidism treatment makes eye symptoms worse?

- Radioactive iodine

Presentation of ischemic hepatic injury?

- Rapid and VERY significant increase in the transaminases with moderate elevations in total bilirubin and alk phos - Diffuse liver injury due to hypotension (look for it after blood loss)

Two major complications untreated hyperthyroid patients are at risk for?

- Rapid bone loss from increased osteoclastic activity - Cardiac tachyarrhythmias, including atrial fibrillation

A 28 year old woman comes to the office due to malodorous vaginal discharge for the last month. LMP 2 months ago, no change in symptoms during that time. Has also had intermittent,crampy abdominal pain, sometimes feels like gas is passing through her vagina. No surgeries. Speculum exam shows malodorous, tan vaginal discharge and a patch of erythema on the posterior vaginal wall. Sinus with purulent drainage is also present in the perianal skin. What is this and what is the most likely underlying disorder?

- Rectovaginal fistula - Crohn's disease - causes fistulas

3 manifestations of CVID?

- Recurrent respiraotry infections (ex. pneumonia) - Sinusitis - GI infections

Timing of MI complications: 1) Reinfarction 2) Ventricular septal rupture 3) Free wall rupture 4) Postinfarction angina 5) Papillary muscle rupture 6) Pericarditis 7) Left ventricular aneurysm

- Reinfarction - hours to 2 days - Septal rupture - hours to 1 week - Free wall rupture - hours to 2 weeks - Posinfarction angina - hours - 1 month - Papillary muscle rupture - 2 days - 1 week - Pericarditis - 1 day - 3 months - Left ventricular aneurysm - 5 days - 3 months

Lithium-induced nephrogenic DI, what part of the kidney affected?

- Renal collecting ducts

73 year old man has symptoms of BPH. Digital rectal exam reveals a smooth and enlarged prostate. His creatinine has increased from 1.2 to 2.1 in four months. Next best step for evaluating this patient's acute kidney injury?

- Renal ultrasound - Most likely has AKI from BPH - need to evalute for hydronephrosis

Managment of hyperosmolar hyperglycemic state?

- Replacement with normal saline - Then IV Regular insulin and potassium repletion only if potassium <5.3

What happens with requirements for levothyroxine in hypothyroidism with estrogen replacement therapy?

- Requirements increase

What are the two indications for initiating home oxygen therapy in patients with COPD?

- Resting arterial oxygen tension less than or equal to 55 mm HG or pulse oxygen saturation less than or equal to 88% on RA - PaO2 less than or equal to 59 or SaO2 les than or equal to 89% with patients wth cor pulmonale, evidence of right heart failure

Explain cor pulmonale

- Right sided heart failure from pulmonary hypertension (common from COPD, interstitial lung disease, pulmonary vascular disease, OSA) - Symptoms of syncope, dyspnea on exertion, fatigue, exertional angina - Peripheral edema, increased JVP, loud S2, right-sided heave, pulsatile liver from congestion, hepatomegaly, possible ascites - chest x-ray may show enlarged central pulmonary arteries

Forest disease that presents with petechial rash?

- Rocky mountain spotted fever - Tx. with doxycycline

What additional heart sound finding can be heard with severe, chronic mitral regurgitation?

- S3

....inhibitors are commonly associated wtih SIADH?

- SSRIs!

Describe calcific uremic arteriolopathy?

- Subintimal fibrosis, calcification of the middle layer of arterioles, and thrombotic occlusion without vasculitis - Associated with end-stage renal disease - Risk factors: hypercalcemia, hyperphosphatemia, hyperparathyroidism, obesity, diabetes, warfarin - Painful nodules and ulcers on the skin - Generally predicts a poor prognosis

Peripheral artery disease treatments.

- Supervisied graded exercise program - Intiate prevention for another cardiac event: aspirin + statin - Doesn't matter if their cholesterol is normal or not - you put anybody with claudication/PAD on a statin

Treatment of orbital mucormycosis?

- Surgical debridement + amphotericin B

Walk thorugh the management of variceal hemorrhage.

- Suspected variceal hemorrhage - place 2 large bore IV catheters - volume resuscitation, IV octreotide, antibiotics - Urgent endoscopic therapy of esophageal varices: - No further bleeding: beta blocker + endoscopic band ligation 1-2 weeks later (secondary prophylaxis) - Continued bleeding: balloon tamponade (temporary) - eventually TIPS or shunt surgery - early rebleeding - repeat endoscopic therapy - recurrent hemorrhage - TIPS or shunt surgery

Man had a DVT. Was discharged on warfarin. Missed an appointment for monitoring because of his schedule. Came back with another DVT. next best step?

- Switch him to something like Rivaroxaban - It is just as effective, but doesn't require monitoring

What is mixed connective tissue disorder and what antibody is associated with it?

- Systemic sclerosis + SLE + polymyositis - Anti-U1 RNP

How does nitroglycerin reduce pain?

- Systemic venodilation which lowers preload and left ventricular end-diastolic volume which reduces myocardial oxygen demand by reducing wall stress

Difference between toxic epidermal necrolysis and stevens Johnson syndrome?

- TEN: usually has other symptoms besides skin such as flu like symptoms; more than 30% of body surface involved - SJS: skin involvement less than 10% of body surface area

What antibiotic can cause hyperkalemia?

- TMP-SMX

Treatment for PJP pneumonia?

- TMP-SMX + corticosteroids

Treatment for nocardia?

- TMP/SMX

What medication is given to all solid organ transplant patients to prevent opportunistic infections?

- TMP/SMX

Treatment for ringworm?

- Terbinafine

26 year old woman with progressive tiredness. Menstrual periods are regular, bleeding lasts 3 days. Hemoglobin 10.1, MCV 70. Fecal occult blood testing negative, iron therapy initiated. 6 weeks later, labs are unchanged. what is going on?

- Thalassemia

Patient with herpes simplex encephalitis is treated wtih acyclovir. Creatinine upon admission was 0.8. Two days later compalins of abdominal pain and nausea. Creatinine is now 2.8. What caused this acute kidney injury?

- The Acyclovir - Causes crystaluria that obstructs the renal tubules

What is pulseless electrical activity and what do you do?

- The presence of an organized rhythm (ex. atrial fibrillation) on cardiac monitorting without a palpable pulse - You start CPR - YOU NEVER SHOCK for PEA - it is not a shockable rhythm

Describe the primary mitral vavle abnormality in patients with hypertrophic cardiomyopathy?

- The presence of systolic anterior motion of the mitral valve, leading to anterior motion of mitral valve leaflets toward the interventricular septum

Why is hypokalemia so difficult to correct in chronic alcoholics?

- They also tend to have hypomagnesemia which is a common and well-known cause of refractory hypokalmemia

Why do empyemas have low glucose concentrations in the pleural fluid?

- They are exudative effusions with a lot of leukocytes and bacteria, which both have high metabolic activity, so they will use all the glucose

Why are multiple myeloma patients at an increased risk for infections?

- They have ineffective antibody production and hypogammaglobulinemia

What diuretics can cause pancreatitis?

- Thiazide diuretics - Furosemide

Bilateral bibasliar lucency?

- Think alpha-1 antitrypsin deficiency

25 year old man, previously healthy. 3 motnhs of progressive dyspnea and nonproductive cough. Feels short of breath when cycling 2 miles to his office, which he used to be able to do without difficulty. No tobacco or alcohol. Works at a publishing firm, no recent travel. Father has asthma. Normal vital signs, normal O2 on RA. Lungs are clear to auscultation. Labs normal. CXR shows mediastinal fullness and scattered reticular opacities in the upper lungs. What are the likely levels (increased, decreased, normal) for FEV1, total lung capacity, and DLCO?

- This is concerning for pulmonary sarcoidosis - typically demonstrates a restrictive pattern - Normal/reduced FEV1, reduced total lung capacity, reduced DLCO

Patient with cirrhosis and kidney injury that does not improve despite volume resuscitation. What is the cause?

- This is hepatorenal syndrome - Causes sphlancnic arterial vasodilation which decreases overall vascular resistance - Activates the RAAS system which causes vasoconstriction and DECREASED RENAL PERFUSION

What are the 5 key features of thrombotic thrombocytopenic pupura?

- Thrombocytopenia, microangiopathic hemolytic anemia, renal insufficiency, neurologic changes ("acting weird"), and fever

Best strategy to prevent progression of chronic kidney disease (both from diabetes and not from diabetes?

- Tight blood pressure control

Treatment for tinea versicolor?

- Topical ketoconazole, terbinafine, or selenium sulfide

Treatment for papulopustular rosacea?

- Topical metronidazole first line - Second line: oral tetracyclines

High T3/4 and thryoxine and low TSH. RAdioactive iodine uptake scan reveals uptake of tracer only in the right thryoid lobe. WHat is this?

- Toxic adenoma - Autonomous thryoid hormoen production without TSH stimulation

For all patients 40+ with diabetes, treat with what medication regardless?

- Treat with a statin always, even if their LDL level is okay - Prevention of cardiovascular disease

Next step in managing a suspected herniated disc?

- Trial of NSAIDS - There really isn't much point to doing an MRI of the spine because it won't change management

22 year old. 4 days of fevere, double vision and painful swelling around his eyes. Significant muscle pain in neck and jaw. Returned from mexico 2 weeks ago. One week after return, had abdominal pain, nausea, vomiting and diarrhea. Symptoms resolved. Has a history of IV drug abuse. Temp is 101, blood pressure and pulse normal. Subungal splinter hemorrhages, periorbital edema, and chemosis. What disease is this?

- Trichinellosis - Acute phase: the abdominal symptoms - Chronic: myositis, fever, subungal splinter hemorrhages, periorbital edema, eosinophilia

Onychomycosis caused by what organism?

- Trichophyton rubrum

Heart murmur most associated with infective endocarditis and what kind of murmur is it?

- Tricuspid regurgitation - Holosystolic murmur that increases with inspiration

What treatment for a patient with non-bleeding esophageal varices?

- Trying to prevent them from bleeding - Can either use endoscopic variceal ligation or a beta-blocker like nadolol or propranolol

What is the single most important prognostic factor in patients with breast cancer?

- Tumor burden aka TNM staging

What two things do white cell casts in urine suggest?

- UTI - Interstitial nephritis

Widespread tinea corporis should raise suspicion for what?

- Underlying immunodeficiency such as HIV - Should test for HIV

What initial anti-coagulant is recommended in patients with PE who have bad renal disease?

- Unfractioned heparin - less renal clearance - Preferred over LMWH, Fondaparinux, Rivaroxaban

54 year old male. signs and symptoms of prostatitis. What next?

- Urine gram stain and cutlure to identify the organism

Guy has left-sided pneumonia. O2 sat is 94% when lying on his right side, but drops to 89% when laying on his right. Explain this.

- V and Q are both lower in the bases of the lungs always - When he lays on his left side, gravity causes more blood to flow into the left lung - There is lower V in that lobe due to the consolidation (V is essentially 0) but now there is increased perfusion so there is now an increased V/Q mismatch (V is 0, Q has increased) - This increased mismatch leads to increased right to left intrapulmonary shunting and increased hypoxemia

Wha t anti-seizure medication is associated wtih drug-induced pancreatitis?

- Valproic acid

For each maneuver, what murmurs get louder and softer: - Valsalva - Standing - Squatting - Handgrip

- Valsava: louder - HCM, MVP; softer - all oterhs - Standing: louder - HCM, MVP; softer - all others - Squatting: louder - AR, MR, VSD; softer - HCM, MVP - Handgrip: louder - AR, MR, VSD; softer - HCM, AS

empiric therapy for endocarditis?

- Vancomycin

What is Ramsay Hunt syndrome?

- Varicella zoster: vesicles in ear, ear pain, ipsilateral facial paralysis

What is mixed cryoglobulinemia?

- Vasculitis - Palpable purura, peripheral neuropathy, liver involvement, arthralgias, and systemic symptoms - Strongly associated with hepatitis C - Low complement level

Villous vs tubular - which one worse?

- Villous adenoma

Compare symptoms of viral esophagitis vs. candida albicancs in patients with HIV?

- Viral esophagitis: severe pain with swallowing as the predominant symptoms, no difficulty swallowing, no thrush - Candida albicans: difficulty swallowing, oral thrush, mild to moderate pain (but not severe)

Picture of circular hair loss patch w/out scales vs w/scales. What is each one and the treatment?

- W/out - alopecia arreata - intralesional triamcinolone - W/scales - tinea capitis - oral griseofulvin (or other antifungal)

What is the most effective nonpharmacologic way to control blood pressure?

- WEIGHT LOSS

What would you expect with urinary bicarbonate excretion and urine ph with chronic respiratory alkalosis?

- Want to get rid of bicarbonate to compensate so... - Increased urinary bicarbonate excretion and increased urinary pH

What anticoagulant leads to low protein S levels?

- Warfarin

What type of tinea fungus is KOH prep good for vs. wood's lamp?

- Wood's lamp:can be good for evaluation of tinea capitis, but not tinea pedis - KOH: good for tinea pedis

What causes fecal incontinence with chronic constipation?

- You can get a fecal impaction - The passage of liquid stool around the impaction can cause fecal incontinence - Common in people who have chronic constipation or impaired mobility

What is Behcet disease?

- Young adults - Recurrent, painful oral aphthous ulcers - Genital ulcers - Eye lesions (ex. uveitis) - Skin lesions - thrombosis

What is the drug of choice for chemotherapy-induced vomiting?

- Zofran - 5HT3 receptor blockers (serotonin receptor antagonists)

Treatment of microadenoma vs macroadenoma in pituitary?

- asymptomatic microadenoma (<10 cm) - no treamtent - Symptomatic micro or macroadenoma: dopamine agonists; if they don't work or greater than 3 cm - surgery

Walk through rabies exposure prophylaxis algorithm.

- bite occurs - Low risk wild animal (squirrel, chipmunk, rabbit) - no PEP - High risk (bat, raccoon, skunk..)- animal available for testing? - yes - euthanize and test, start PEP if positive; not available for testing - start PEP - Pet (dog, cat) - available for quarantine - no - start PEP; yes - observe for 10 days - no pep if animal is healthy

What medication to use for cancer induced cachexia vs. HIV cachexia?

- cancer induced: megestrol acetate - HIV: dronabinol (synthetic cannabinoids)

Clostridium septicum bacteremia is strongly associated with?

- colon cancer

Megaloblastic anemia with phenytoin use. Whats the cause?

- folic acid deficiency

UA in ethylene glycol poisoning?

- rectangular, envelope-shaped calcium oxalate crystals

What is vasospastic (Prizmental) angina?

- recurrent episdoes of chest discomfort that resolve within usually 15 minutes - commonly occur during sleep and may be accompanied by diaphoresis, nausea, palpitations and dyspnea - Typically younger patients who lack classical risk factors for coronary artery disease - ST elevations only during pain attack - Treat with calcium channel blocker (preventative) and nitrates (abortive)

"Curtain coming down over my eyes."

- retinal detachment

Secretory vs osmotic diarrhea?

- secretory: occurs due to toxins, hormones (produced by like VIPomas), congenital disorders of ion transport (cystic fibrosis), or bile acids (in postsurgical patients) - LOW stool osmotic gap - osmotic: due to the presence of a nonabsorbed, osmotically active solute (polyethylene glycol, sorbitol, lactose), which inhibits water reabsorption; occurs after ingestion of a causative substance - HIGH stool osmotic gap

Equations for sensitivity and specificity?

- sensitivity: TP/TP + FN - Specificity: TN/TN + FP

Dyspnea with elevated BNP. What other finidng and what diagnosis?

- third heart sound - Heart failure

Common side effect of pembrolizumab?

- thyroid dysfunction

Initial treatment for plaque psoriasis?

- topical steroids

What is teh definition of hypoxemia for somebody on a ventilator? What should you do to improve this?

- PaO2 <60 - You need to get it up and you can do this two ways: increase FIO2 or increase PEEP - However, usually these people are already on started on high amounts of FIO2 and you don't want to increase it anymore - you want to try to get FiO2 down to <60% as quicly as possible because prolonged high FiO2 increases the risk of pulmonary oxygen toxicity - So instead, increase the PEEP to improve the hypoxemia

Epididymitis organisms based on age.

- >35: usually E. coli - <35: usually STD: chlyamydia or gonorrhea

Joints affected in rheumatoid arthritis vs osteoarthritis?

- RA: MCP mainly - OA: DIP and PIP

Explain a relative risk of 0.71 in a trial where they were trying to see if aspirin decreased CV events.

- A person taking aspirin as 0.71 the risk of a CV event as a person who is not taking aspirin

What medications have been shown to improve long-term survival in patients with left ventricular systolic dysfunction

- ACe inhibitors - ARBs - Beta blockers - Aldosterone antagonists

Early onset hypertension (ex. young people) + bilateral upper abdominal masses?

- AD polycystic kidney disease - Diagnosed wtih abdominal ultrasound

Name the metabolic and electrolyte side effects associated with thiazide diuretics?

- Hyperglycemia, increased LDL and increased triglycerides, and hyperuricemia - Hyponatremia, hypomagnesemia, hypokalemia, and hypercalcemia

What is a common cause of hypocalcemia in alcoholics?

- Hypomagnesemia - Induced resistance to PTH - Cause of low magnesium in alcoholics is multifactorial

Findings in Goodpasture syndrome?

- AKA anti-basement membrane disease - Renal findings such as acute renal failure and dysmorphic red cells and red cells casts in urine - Pulmonary findings including shortness of breath, cough and hemoptysis

Attributable risk calculation?

- AR = (RR-1)/RR

What cardiac complication can occur from Lyme disease?

- AV block

25 year old woman. occasional episodes of palpitations. unprovoked, unrelated to exertion. one episode, heart rate was 185 and regular. reports that self-immersing her face in cold water relieves symptoms very quickly. What is this?

- AV nodal reentrant tachycardia - Most common form of PSVT, often develops in young patients with a structurally normal heart

Filamentous gram-positive rods with rudimentary branching. treatment?

- Actinomyces - Dental infections and trauma - Penicillin

53 year old woman. right sided abdominal pain that started 2 days ago. severe enough to interfere with sleep. No vomiting/diarrhea/nausea .Had breast cancer diagnosed a year ago, completed last course of chemo 3 weeks ago. Normal vital signs. Normal abdominal exam with non-distended, no rebound, guarding or rigidity. Lightly brushing the skin to the right of the umbilicus elicits intense pain. What is this?

- Actually herpes zoster AKA shingles - Pain may precede the rash by severeal days - Frequently triggered by severe stress (such as cancer treatment in this patient)

Explain the different types of tranfusion reactions: - Acute hemolytic tranfusion reaction - Febrile non-hemolytic tranfusion reaction - Minor allergic reaction - Anaphylaxis - TRALI - Delayed hemolytic tranfusion reaction

- Acute hemolytic: caused by ABO incompatibility; happens immediately; immediately stop tranfusion; prevent by repeated typing and cross-matching of blood products - fever, flank pain, etc. - Febrile non-hemolytic: within 1-6 hours; fevers, chills, malaise; treat wiht acetaminophen; using leukoreduced blood products - Minor allergic reaciton: urticaria during tranfusion; pretreat with diphenhydramine - Anaphylaxis: anti-IgG IgG in recipienets; sudden onset during the transfusion; shock, hypotension; epinephrine, hemodynamically stabalize, secure airway - TRALI: soluble factors in donor blood leads to activation of recipient's granulocytes; during or within 6 hours of transfusion; syptoms and imaging generally teh same as in ARDS; discontinue tranfusion, supportive managmenet - Delayed: days or weeks after; occurs in patients who were previously sensitized to specific RBC antigens during transfusions, pregnancy or transplanatations; fever, jaundice, anemia, dark urine

What is scleroderma renal crisis and how do you treat it?

- Acute onset of hypertension and acute kidney injury in patients with systemic scelrosis - Treat with ACE inhibitors is first line

Lymphocytic infiltration of the intima of the kidney?

- Acute rejection

60 year old asymptomatic man with T2DM. He has microalbumineria. What should you do to reduce end-organ damage?

- Add an ACE inhibitor - Ace inhibitors slow progression of diaphretic nephropathy

How does pharmacologic stress testing with drugs such as adenosine help to reveal ischemic heart disease?

- Adenosine causes vasodilation and increased myocardial blood flow - Several-fold augmentation of blood flow in non-obstructed coronary artreies, and blood flow is increased in stenosed coronary arteries as well, but to a much lesser extent. - The relative blood flow difference is magnified, causing a detectable reduction in radioactive isotope uptake by myocardial cells in areas supplied by a stenotic artery - appears as an ischemic defect on myocardial perfusion imaging

Define adjuvant, consolidation, induction, maintenance, and neoadjuvant therapy for cancer.

- Adjuvant: treatment given in addition to standard therapy - Consolidation: typically given after induction therapy with multidrug regimens to further reduce tumor burden - Induction: initial dose of treatment - Maintenance therapy: usually given after induction and consolidation therapy - Neoadjuvant: treatment given before the standard therapy for a particular disease

Who do we vaccinate against Hepatitis A?

- Adults who are at an increased risk for infection such as men who have sex with men and traveleres to countries where hep a is prevalent - hepatitis A became a routine part of childhood vaccines in 2006 so keep that in mind when you're looking at the age of the patient

Walk through the management of minimal bright red blood per rectum.

- Age <40 without red flags: anoscopy - either hemorrhoids (don't need to do anything else) or no source identified - then need to do sigmoidoscopy or colonoscopy - Age 40-49 without red flags: sigmoidoscopy or colonscopy - Age 50+ or red flags: colonoscopy

Treatment of hyperosmolar hyperglycemic state?

- Aggressive hydration with normal saline - IV insulin - Potassium replacement if level is <5.3

Blowing and high pitched holosystolic murmur heard best over the apex?

- Mitral regurgitation

Most comon valvular dysfunction in IE NOT from drug use?

- Mitral regurgitation

Holosystolic murmur loudest at the apex and radiates to the axilla? What is the cause?

- Mitral regurgitation - In developed countries: caused by mitral valve prolapse usually - mxomatous degeneration of the mitral valve leaflets and chordae

Early diastolic sound followed by middiastolic murmur?

- Mitral stenosis - Opening snap

What is the best test to determine the risk of foot ulcer in a diabetic patient?

- Monofilament test

Treatment for chronic pancreatitis?

- Alcohol and smoking cessation - Pancreatic enzyme supplementation - Frequent, small meals

How do alkalosis and acidosis affect serum ionized calcium?

- Alkalosis - elevated blood ph - dissociation of hydrogen bound to albumin, leading to increased calcium binding and a decrease in serum calcium - Acidosis - decreased blood ph - increased binding of hydrogen bound of albumin, leading to decreased calcium binding and an increase in serum calcium

Sildenafil plus what other category of drug combined can cause severe hypotension?

- Alpha blocker such as doxazosin

Describe Osler Weber Rendu syndrome.

- Also called hereditary hemorrhagic telangeictasia syndrome - Diffuse telangiectasias (rub-colored papules that blanch with pressure), recurrent epistaxis, and widespread AVMs - AVMs in the lungs can shunt blood from right to left side of the heart, causing chronic hypoxemia, digital clubbing, and a reactive polycythemia - Pulmonary AVMs can sometime presents as massive, fatal hemoptysis

What is age-related sicca syndrome?

- Also known as age related dry eye syndrome - Exocrine output from lacrimal and salivary galnds declines with age - Complain of ocular discomfort, can develop caorneal erosions, and have impaired vision

What medication should you use for lyme disease in pregnant women/lactating women and children?

- Amoxicillin - You can't use doxycycline - permanent discoloration of teeth

24 year old man. dyspnea on exertion. chronic low back pain. smokes 1 pack per day. vital capacity 75% of predicted, FEV1/FVC 95%, FRC 110%. What is this?

- Ankylosing spondylitis - Causes back pain and chest wall motion restriction

Anterior uveitis vs episcleritis?

- Anterior uveitis: painful, red eye with tearing and decreased visual acuity - Episcleritis: acute redness and tearing; mild irritation, but no overt pain and diminshed visual acuity

What antibodies associated wtih systemic sclerosis?

- Anti-topoisomerase 1

Widening of the mediastinum?

- Aortic dissection

Descrescendo early diastolic murmur?

- Aortic regurgitation

Delayed carotid pulse, what problem?

- Aortic stenosis

Harsh systolic murmur heard best at the right second intercostal space with radiation along the carotid arteries?

- Aortic stenosis

Arterial embolism vs. arterial thrombosis vs. venous thrombosis physical exam findings.

- Arterial embolsim: sudden and severe pain, cold and pale limb, emboli typically originate from heart, unilateral pulselessness on the affected side - Arterial thrombosis: slow, progressive narrowing; pulses are usually diminished bilaterally - Venous thrombosis: pain and edema but more dull, achy pain; can still be warm; no pulselessness

Inflammatory bowel disease is frequently complicated by?

- Arthritis

Patients presenting to the ED with chest pain and suspected acute coronary symptoms should be treated with what?

- Aspirin - Even if you have low suspicion for an actual cardiac event, as long as you don't htink they're dissecting, you should give them aspirin - It won't hurt them and it does have significant benefit if they are having an actual cardiac event

What arrhythmia is most specific for digoxin toxicity?

- Atrial tachycardia with AV block

78 year old man. episode of syncope. EKG shows sinus bradycardia. What next?

- Atropine - If this doesn't work, can try IV epinephrine or transcutaneous pacingd

What cause orthostatic hypotension in Parkinsons'?

- Autonomic dysfunction

Inheritance pattern on hypertrophic cardiomyopathy?

- Autosomal dominant

Other than antivirals, what other treatments when soembody has chronic hepatitis C?

- Avoidance of alcohol - Hepatitis A and B vaccination

Why do patients with nephortic syndrome have hypocalcemia?

- B/c they have low albumin - decreased total serum calcium - however, ionized calcium (physiologically active form) is hormonally regulated and remains normal

Presentation of each: - Babeiosis - Rocky mountain spotted fever - Ehrlichiosis - Lyme disease

- Babesiosis: flu-like symptoms, intravascular hemolysis and thrombocytopenia - Rocky mountain spotted fever: fever, headache, myalgias, and prominent rash - Erhlichiosis: fever, malaisea, headache, leukopenia/thrombocytopenia - Lyme disease: rash is big one

How does dobutamine work in decompesnated heart failure and cardiogenic shock?

- Beta 1 agonist - Increases cAMP in cardiac myocytes which increases cardiac myocardial contractility - Increased contractility pushes more blood out so left ventricular end-systolic volume is decreased

What common class of drugs can cause weight gain and worsening insulin resistance?

- Beta blockers

What 3 classes of drugs prevent remodeling and reduce mortality in pateints after MI?

- Beta blockers - ACEi - Aldosterone antagonists

What 2 treaments for hypertrophic cardiomyopathy and presentation of it?

- Beta blockers and calcium channel blockers - Asymptomatic or runs in families - Fatigue, chest pain, palpitations, syncope - Sudden syncope during athletics especially in young athletes

Chest x-ray findings for PJP pneumonia? Treatment?

- Bilateral nterstitial infiltrates - TMP-SMX +/- steroids - Use steroids when: PaO2 less than or equal to 70, A-a gradient greater than or equal to 35, pulse oximetry < 92% on RA

Treatment of Paget's disease?

- Bisphosphonates

Heaped-up skin lesiosn with a violaceous hue and sharply demarcated border. Dry cough, mild malaise. What fungal infection is this?

- Blastomycosis

CMV retinitis in HIV patients presentation?

- Blurred vision, floaters and photophobia - Yellow-white, fluffy, hemorrhagic lesions

How to differentiate between acute transplant rejection and bacterial infection of a liver transplant?

- Both can have RUQ pain, elevation in liver enzymes and fever - High fever, significant leukocytosis and hemodynamic instability are more concerning for bacterial infection

Rapidly stop corticosteroids. What happens to ACTH and cortisol?

- Both will be decreased - Steroids suppress ACTH production by the pituitary and cortisol production by the adrenal glands

Why can a PE cause hypocalcemia?

- Breath faster = respiratory alkalosis - Causes dissociation of hydrogen ions from albumin, allowing increased binding of calcium, leading to a drop in unbound calcium

Linear IgG and C3 deposits at the dermal-epidermal junction? Treatment?

- Bullous pemphigoid - First line: topical steroids (clobetasone)

Peripheral edema is a common side effect of what class of anti-hypertensives?

- Calcium channel blockers

Radiographic appearance, cause, and crystal shape of each: - Calcium oxalate - calcium phosphate - Struvite - Uric acid

- Calcium oxalate: high-sodium, high-oxalate diet, hyperparathroidism; small, radiopaque; octahedron or envelope - Calcium phosphate: hyperparathyroidism, renal tubular acidosis; small, radiopaque; wedge or rosette - Struvite: upper tract infection with urease-producing organism (proteus); large, radiopaque; rectangular/prism - Uric acid: gout, diabetes, myeloproliferative disorders; small, radiolucent; yellow brown and rhomboidal (prevent with urine alkalanization - aka potassium citrate)

Effect of isoniazid on the liver and what to do about it?

- Can cause INH hepatitis - can be severe, if seems severe, stop the medication immediately - More common - 10-20% of patients will have mild, subclinical hepatic injury demonstrated only by mild elevations in serum aminotranfererases (<100) - prognosis is excellent in these patients and condition is self-limited, allowing for INH therapy to be maintained with close follow-up

Patient with COPD exacerbation started on supplemental oxygen. Becomes confused, lethargic, and has a seizure. Why?

- Carbon dioxide retention - Supplemental oxygen can worse hypercapnia in COPD patients due to a combination fo increased dead space perfusion causing ventilation-perfusion mismatch - Increase in CO2 can cause a change in the level of consciousness - hypercapnia can also cause cerebral vasodilation and may induce seizures

Case control study and what risk/ratio/etc. are you reporting?

- Case control: two groups, one already has the outcome, one doesn't; looking back to see exposures in the past - Report exposure odds ratio - Vs. cohort study you report relative risk

What is Chagas disease?

- Caused by Trypanosoma cruzi - Common throughout latin america - Two primary manifestations: megacolon or megaesophagus + cardiac disease

Describe central DI vs nephrogenic DI.

- Central DI: caused by decreased ADH release from the pituitary ; can be idiopathic, trauma, pituitary surgery, or ischemic encephalopathy; HIGH serum Na - Nephrogenic DI: caused by ADH resistance in the kidney; caused by chronic lithium use, hypercalcemia, and hereditary; NORMAL serum Na

Loss of vision in eye for five minutes. Similar episode earlier in the day. Visual acuity of 20/60 and subtal retinal whitening. What is this, what is the next step?

- Central retinal artery occlusion - Medical emergency: ocular massage and high flow oxygen

What cerebral complication with polycystic kidney disease?

- Cerebral aneurysms

Explain euthyroid sick syndrome.

- Characterized by low T3 levels with normal TSH and T4 in patients with acute illness - Due primarily to decreased peripheral conversion of T4 to T3 - Treatment is generally not recommended unless abnormal thyroid function persists after the patient has returned to baseline health

Calcification of articular cartilage think of what?

- Pseudogout

26 year old man with urethral discharge. Gram stain and cutlure do not show any bacteria or growth. What is teh most likely organism?

- Chlamydia - Gonorrhea grows in 95% of cases

Most frequent location of origination of atrial fibrillation?

- Pulmonary veins

What is angiodysplasia?

- Dilated submucosal veins and AV malformations - Can occur anywhere in the GI tract but most common in the right colon - Common cause of recurrent painless GI bleeding

24 year old woman. Lost 11 lb. Has orthostatic vital signs positive. Looks dry. Labs show low serum sodium, potassium and chloride. High urine potassium and high urine sodium. What is the most likely cause?

- Diuretic abuse - Normally with hyponatremia, you have low urine sodium - but not if you're using diuretics - Normally with hypokalemia - patients respond by reducing urinary potassium excretion - but once again, not with diuretic use

Low urinary excretion after D-xylose. Treat with rifaximin. Does fix it vs. doesn't fix it.What diseases?

- Does fix: small intestinal bacterial overgrowth - Doesn't fix: somethingn like celiac disease

Most common cause of bloody diarrhea in the absence of fever?

- E. coli

Pathophysiology of heat exhaustion vs. exertional heat stroke

- EXertional heat stroke: thermoregulation failure - Heat exhaustion: inadequate fluid and salt replacement

What testing should be performed before starting trazustumab?

- Echo - Cardiotoxicity is a known adverse effect of this drug

What are the 3 requirements for acute liver failure?

- Elevated aminotransferases (often >1000) - Signs of hepatic encephalopathy - Impaired hepatic synthetic function (defined as INR greater than or equal to 1.5)

Symptoms of diffuse esophageal spasm?

- Episodes of chest pain, difficulty swallowing liquids and solids during these episodes of apin

Coccidiodes causes what classic skin manifestation?

- Erythema nodosum

Symmetric circumferential narrowing affecting the distal esophagus?

- Esophageal stricture

Chronic mesenteric ischemia: etiology, clinical features, diagnosis, and managemnet

- Etiology: atherosclerosis (smoking, dyslipidemia) - Clinical features: crampy, postprandial epigastric pain; food aversion; weight loss - Diagnosis: signs of malnutrition, abdominal bruit, CT angiography - Management: risk reduction, nutritional support, endovascular or open surgical revascularization

Patient with peripheral arterial disease. Started on aspirin and statin. What else?

- Exercise therapy

Explain Light's criteria and what each type is caused by.

- Exudative if any of the following: 1) Protein (pleural/serum) >0.5 2) LDH (pleural/serum) > 0.6 3)pleural LDH > two-thirds upper limit of normal serum LDH - Exudative: infection, malignancy, PE - Transudative: hypoalbunemia (cirrhosis,nephrotic syndrome), congestive heart failure

3 main causes each of exudative and transudative pleural effusion

- Exudative: infection, malignancy, rheumatologic disease - Transudative: heart failure, cirrhosis, nephrotic syndrome

First line treatment for open-angle glaucoma?

- Eye drops - latanoprost, bimatoprost - If these are ineffective - topical beta blockers like timolol can be added

Most common cause of unprovoked DVT in a young, healthy white patient with no risk factors?

- Factor V leiden: activated protein C resistance

RA + splenomegaly + neutropenia - what disorder?

- Felty syndrome

Bilateral, nonfocal chest tenderness and diffusely nodular breasts. 25 year old woman, recently stopped taking oral contraceptives. What is this?

- Fibrocystic changes (benign) - Usually oral contraceptives can help the symptoms, so since she stopped taking them she is noticing it

Diaphragmatic flattening with prominent bronchovascular markings vs. decreased markings?

- Flattening + increased markings - bronchitis - Flattening + decreased markings - COPD

Treatment of choice for SIADH in asymptomatic or mildly symptomatic patients?

- Fluid restriction

Management of SIADH?

- Fluid restriction +/- salt tablets - Hypertonic (3%) saline for severe hyponatremia

What is Herpes Simplex keratitis?

- Frequent cause of corneal blindness in the US, usually in adults - Complaints of pain, photophobia, blurred vision, tearing and redness - Corneal vesicles + dendritic ulcers

Waht is the treatment/screening for FAP and waht are indications to do something different?

- Frequent colonoscopic surveillance - Indications for immediate surgery: patients who present with colorectal cancer, adenomas with high-grade dysplasia, severe symptoms from it (ex. hemorrhage), significant increase in polyp number between screening interval

Characerisitics of laxative abuse?

- Frequent, watery, nocturnal diarrhea - Colonscopy with characersitic findings of melanosis coli - dark brown discoloration with pale patches of lymph follicles

What class of diabetes medications can help with weight loss?

- GLP-1 agonists

Compliance, gas exchange and pulmonary arterial pressure with ARDS?

- Gas exchange impaired - Lung compliance decreased - Pulmonary arterial pressure increased

A 28 year old woman comes to the office with acue pain in her left knee joint. Mild discomfort and pain in her wrist 4 days ago. Left ankle pain 2 days ago. No medical problems. No drug use. SExually active. Temp 101.3. Knee has moderate effusion and faint erythema. Warmth and tenderness on palpation. Range of motion limited by pain. Synovial fluid analysis shows white blood cell count of 50,000 with a predominance of neutrophils. most likely cause?

- Gonococcal septic arthritis - Septci arthritis is likely to be gonococcal (not non-gonococcal) in a young, sexually active person

Treatment of gonorrhea? Treatment of chlamydia?

- Gonorrhea - azithromycin + ceftriaxone - Chlamydia - only need azithromycin - If you can't rule out chlamydia - you HAVE to treat empirically with both

Good vs poor demarcation on erythematous face rash?

- Good demarcation - erysipelas - Group A strep - Poor demarcation - cellulitis - Group A strep, MSSA, MRSA

What is considered a positive TB skin test in patietns wtih HIV?

- Greater than 5 mm - Treat with isoniazid (and pyridoxine) for 9 months

Case control study?

- Group already has the outcome; look back and look for the exposure

Development of MALT lymphoma is classically associated with what?

- H. pylori infection

most appropriate testing to diagnose acute hepatitis B infection?

- HBsAg and IgM anti-HBc

Sudden onset severe psoriasis think what other disease?

- HIV

Widespread mollucsum contagiosum is associated wtih what disease?

- HIV

Granulomas with narrow-budding yeast?

- HIstoplasmosis

Plantar/flat warts caused by what?

- HPV - remmeber, benign and malignant strains!

Peripheral blood smear that reveals bite cells and red blood cell inclusions on crystal violet staining. What is this/disease?

- Heinz bodies and bite cells - G6PD deficiency

64 year old male. sharp left-sided chest pain for the last 24 hours. worsened by deep breathing and relieved wtih leaning forward. T2DM, hypertension, hyperlipidemia, hypothyroidism, chronic kidney disease. Friction rub is heard at the left lower sternal border. ECG shows normal sinus rhythm with asymmetric T wave inversion in leads V5 and V6. BUN 96, creatinine 5.1 What therapy?

- Hemodialysis - This is uremia pericarditis - occurs in 6-109% of renal failure patients, typically with BUN > 60 - Hemodialysis is the best treatment

Postpartum patient comes in with moderate respiratory distress and symptoms consistent with a PE. Chest x-ray does not show any infiltrates. Next best step?

- Heparin infusion - You give heparin before doing further diagnostic testing if there is a high probability that the person has it - If the probabilty seemed lower, you would do CT angiography first before treating

What blood thinner during pregnancy?

- Heparin or LMWH NOT warfarin

Patient with findings suspicious for PV with abdominal pain. What is probably going on and what test should you do first?

- Hepatic vein thrombosis from being hypercoagulable - Do a abdominal Doppler ultrasound

What is the most common vaccine-preventable disease among travelers to developing countries?

- Hepatitis A

Porphyria cutanea tarda frequently associated wtih what other disease?

- Hepatitis C

Patient being treated for TB. Tender hepatomegaly, scleral iceterus. high AST and ALT. Liver biopsy shows panlobular mononuclear infiltration and hepatic cell necrosis. What is this?

- Hepatitis from isoniazid

Walk through evaluation of hyperthyroidism.

- High TSH, high free T3/T4 - secondary hyperthyroidism - MRI of pituitary gland - Free T3 and T4 high, TSH low - primary hyperthyroidism - then signs of Grave's disease (goiter + ophthalmopathy)? - Yes? - Graves disease - No? - Do radioactive iodine uptake scan - High uptake: diffuse pattern = Graves disease; nodular pattern = toxic adenoma, multinodular goiter - Low uptake: measure serum thyroglobulin - High thyroglobulin: thyroiditis, iodide exposure - Low thyroglobulin: exogenous hormone

39 year old woman comes to office with AUB. underwent menarche at age 15 and had irregular menses that normalized wtih OCPs. menses have become increasingly irregular and now occur every 2-3 months with heavier bleeding and passage of clots. difficulty concentrating at work due to irritability and depressed mood. No headahces or hot flahses. BMI 26. skin is cool and dry. pelvic exam recveals well-rugated vagina, mobile uterus and small, nontender ovaries. Pregnancy test negative. Levels of TSH, FSH and prolactin?

- High TSH, high prolactin, low FSH - This is hypothyroidism - low T3/4 means no feedback so high TRH/TSH - which activates prolactin causing high prolactin - Low FSH because prolactin inhibits the hypothalmaus and pituitary

Levels of calcium, phosphate and potassium with tumor lysis syndrome?

- High phosphate and high potassium - released from cells - Phosphate binds calcium - LOW calcium

55 year old woman. 2 years of episodic cough productive of yellow sputum. First episode lasted nearly 3 weeks with chest congestion, cough productive of purulent sputum, and SOB. Diagnosed with bronchitis at that time. Condition improved after antibitoics. She has since had 6 more episdoes, each with cough productive of large amounts of yellow sputum, SOB, and sinus congestion. Every time, improves with antibiotics. However, continues to have a productive cough. Never smoked. Vital signs normal. Lung exam reveals rhonchi and wheezes with coarse crackles b/l in the bases. CXR reveals linear atelectasis in the mid and lower lung fields bilatearlly. What is the best step for confirming the diagnosis in this patient?

- High resolution CT scan of the chest - This is bronchiectasis

White casts in the urine? (2)

- Pyelonephritis or interstitial nephritis

Patient with presumed sarcoidosis is given immunosuppressive therapy (Ex. steroids) but deteriorates rapidly. What is this?

- Histoplasmosis - Should be considered if you think its sarcoid but they get worse after treatment

Worsened chest pain with exposure to alcohol, large anterior mediastinal mass. What is this?

- Hodgkin lymphoma

Treatment for primary adrenal insufficiency?

- Hydrocortisone - Can add fludrocortisone if necessary (usually have to)

Dilated esophagus on esophagram, may see narrowing towards the bottom?

- "Bird-beak narrowing" - Esophageal achalsia

What is considered a positive induation size for tuberculin skin test?

- 15 or greater mm

How many drugs in post-exposure HIV prophylaxis?

- 3 drugs - Start immediately

Explain the pneumococcal vaccine regimens for patient.

- 65 and older: 13 and then 23 like a year later - less than 65 with sickle cell, asplenia, immunocompromised, cochlear implant, cerebrospinal fluid leaks - 13 and then 23 - less than 65 with other chornic medical conditions (not listed above) - only 23 and then once they are 65, you would do 13 and 23

Colon cancer screening recommendations for IBD (Crohn's and ulcerative colitis?)

- 8-10 years post diagnosis (12-15 years if disease only in left colon) - Repeat every 1-3 years

What is considered a positive response to bronchodilators?

- >12% increase in FEV1

34 year old female. fatigue for several months. Difficulty combing hair as she can't hold her hands over her head for a long period. Anxiety, irritability, unintentional weight loss fo 8 lb over the last 2 months. Gait is normal, but patients awkwardly drops into a chair when asked to slowly sit. Appears to have decreased muscle mass in her shoulders, but muscles are nontedner. What is this?

- Chronic hyperthyroid myopathy - A muscle illness such as polymyositis could also cause similar symptoms, but wouldn't have all the other symptoms she does of hyperthyroidism

Signs/symptoms of arsenic poisoning?

- Chronic: Stocking-glove neuropathy; hypo-hyperpigmentation, hyperkeratosis - Acute: garlic breath, vomiting, watery diarrhea, QT prolongation

Main side effects of the following drugs: 1) Cisplatin, carboplatin 2) Ethmabutol, hydroxychloroquine 3) Phenytoin, isoniazid 4) Amiodarone, lithium 5) Cyclophosphamide

- Cisplatin, carboplatin: cochlear dysfunction - Ethambutol, hydroxychloroquine: optic neuritis - Phenytoin, isoniazid: peripheral neuropathy - Amiodarone, lithium: thyroid dysfunction - Cyclophosphamide: acute cystitis, bladder cancer

Bleeding pattern of each: - Colonic angiodysplasia - Diverticulosis - Colon cancer - Hemorrhoids - Ischemic coliti s - Acute mesenteric thrombosis - Peptic ulcer disease

- Colonic dysplasia: painless bleeding in the right colon, but much less comon than diverticulosis - Diverticulosis: most common cause of gross lower GI bleeding in adults, typically painless - Colon cancer: Chronic acult blood loss, not usually gross - Hemorrhoids: painless rectal bleeding, but just on toilet paper or in toilet bowl, not usually a huge gross amount - Ischmic colitis: sudden onset of abdominal pain, followed by rectal bleeding - Acute mesenteric thrombosis: abdominal pain out of proportion to findings, N/V, bloody diarrhea - Peptic ulcer disease: blood gastric aspirate

Diagnostic paracentesis fluid analysis for new-onset ascites includes what tests?

- Color, total protein count, serum-ascites albumin gradient - Also do cell count and differential to rule out spontaneous bacterial peritonitis

Most common cause of medical errors?

- Communication failures between providers

echo findings in acromegaly?

- Concentric left ventricular hypertrophy

Chest x-ray that shows "spotty calcifications along the left heart border."

- Constrictive pericarditis

A 36 year old man comes to urgen care for 2 day history of rash on hand. Small blisters with oozing yellow fluid. Severe itching. Cleaned bushes around his house 2 days ago. Medical history unremarkable. Skin examination shows erythematous plaques with eroded vesicles and small bullae between the fingers of his left hand and dorsum of his right hand. Culture of the vesicular fluid grows coagulase-negative staph. What is the most likely diagnosis?

- Contact dermatitis (poison ivy) - Don't get confused by the Staph growing - this is a common contaminant in wounds but a true infection is rare in healthy patients

Patient with painful red eye, opacification and ulceration of the cornea, wears contact lenses. what is it and what organisms?

- Contact lens-associated keratitis - Most common Pseudomonas and Serratia, but can also be gram-positive organisms as well as certain fungi and amoebas - Medical emergency - need to remove and discard contact lenses, broad spectrum antibiotics

Eye problem associated wtih Sjogren disease?

- Corneal ulceration/damage

Elevated PT and/or PTT - do inhibitor mixing study - what does each result mean?

- Corrects: factor deficiency - Doesn't correct: inhibitor likely present; test for coagulation factor inhibitors

Symptoms of superior pulmonary sulcus tumor?

- Cough, weight loss, lymphadenopathy - Arm/shoulder pain

Empirically treat for what poisoning in patients with smoke inhalation/fires?

- Cyanide poisoning

What is sympathetic ophthalmia?

- Damage of one eye (the sympathetic eye) after a penetrating injury to the other eye - Due to an immunologic mechanism involving the recognition of "hidden" antigens

What is obesity hypoventilation syndrome?

- Daytime hypercapnia (CO2>45) in an obese patient without another epxlanation for the hypercapnia - Most patients have coexisting obstructive sleep apnea with frequent apenic events and daytime hypersomnolence

What causes orthostatic hypotension as people get older?

- Decreased baroreceptor responsiveness

cardiac index, SVR and LVEDV in heart failure?

- Decreased cardiac index - Increased SVR - Increased left ventricular end-diastolic volume

Delayed sleep-wake disorder vs advanced sleep wake disorder vs shift work disorder

- Delayed sleep-wake phase disorder: people who are "night owls"; they consistently can't fall asleep at a normal time (ex. before midnight) so if they have to get up early for something, they physcially can't function - Advanced sleep-wake phase disorder: basically the opposite; can't stay awake past 7 pm and wake up very early - Shift work disorder: recurrent pattern of sleep interrution due to shift work; ex. a person who has a normal internal circadian rhythm who is required to work the night shift

AA amyloidosis of the kidneys occurs most commonly from what other disease? What would you see with kidney biopsy?

- Develops often in the setting of chronic inflammation - most commonly RHEUMATOID arthritis - Kidney: congo-red positive glomerular deposits

Claw toes in a patient with diabetes. What is the cause?

- Diabetic neuropathy

What kind of murmurs are always pathologic even without any symptoms and require further workup?

- Diastolic and continuous murmurs

Patient with symptoms of heart failure. Echo shows normal-sized left ventricle with left ventricular hypertrophy, normal EF. What is this?

- Diastolic dysfunction

What can happen to extremities with high dose norepinephrine?

- Digital necrosis from medication-induced vasoconstriction

What changes to the heart does viral myocarditis cause?

- Dilated cardiomyopathy

16 year old boy diagnosed wtih acute lymphoblastic leukemia. Started on IV fluids, allopurinol and combination chemo. The next day, he develops nausea. Cardiac monitoring shows premature ventricular beats. Repeat labs showhigh leukocytes, high BUN, high creatinine, and very high lactate dehydrogenase. What are the values of calcium, phosphate, and potassium

- Increased phosphorous and potassium - Decreased calcium - This is tumor lysis syndrome: initiation of cytotoxic chemotherapy; acute kidney injury, severe electrolyte abnormalities, cardiac arrhythmias

A 54 year old woman comes to the ED due to severeal hours of severe epigastric abdominal pain. The patient began having mild colicky upper abdominal pain after dinner last night and took Tylenol before going to bed. In the night, the patient awoke with severe abdominal pain radiating to her back. She has also had nausea and several episodes of vomiting. The patient has a history of occasional upper abdominal pain after eating but has never had such severe symptoms. She has no other medical issues and takes no medications. Temperature is 99.6, blood pressure is 110/66, pulse is 118 and respirations are 24. The patient appears to be in moderate distress. The abdomen is distended with marked epigastric tenderness. Bowel sounds are decreased. Lab results show elevated lipase. What are the levels of renin, efferent arteriolar resistance, and tubular sodium reabsorption going to be (increased/decreased)?

- Increased renin, increased efferent arteriolar resistance, and increased tubular sodium reabsorption - This is acute gallstone pancreatitis - She became hypovolemic from severe vomiting, leading to the kidneys increasing renin release - Renin converts aldosterone to angiotensin I, which is then converted to angiotensin II by ACE - Angiotensin II causes vasoconstriction, sodium and water resorption and aldosterone secretion

Cause of Menniere's disease?

- Increased volume or pressure of endolymph

Measures to prevent urinary calcium stone formation?

- Increasing fluid intake - Following a low sodium, low protein diet - Maintaining moderate calcium intake - Taking thiazide diuretics

How does sample size affect confidence interval?

- Increasing the sample size decreases the width of the confidence interval

What is dacryocystitis?

- Infection of the lacrimal sac - Sudden onset of pain and redness in the medial canthus - Sometimes a purulent discharge is able to be expressed

Initial treatments when somebody comes in in diabetic ketoacidosis.

- Infusion of normal saline - Add dextrose 5% when serum glucose is less than 200 - Start continous IV insulin infusion (hold if potassium < 3.3) - Add IV K if serum <5.3

Signs/symptoms of ethylene glycol poisoning?

- Initial symptoms can mimic ethanol inebriation - However, as alcohol dehydrogenase metabolizes ethylene glycol to oxalic acid and glycolic acid, these metabolites cause profound clinical consequences - Glycolic acid damages the renal tubules, and oxalic acid binds calcium, causing hypocalcemia and calcium oxalate crystal deposition in the kidneys - Patients develop flank pain, hematuria, oligurai, acute kidney injury and anion gap metabolic acidosis - Remember, treat with fomepizole

What is the fastest way to rapidly lower serum potassium levels?

- Insulin and glucose

Abdominal/bladder pain that is relieved with urination?

- Interstitial cystitis

Lung problem with systemic sclerosis?

- Interstitial lung disease

What is a factorial design study?

- Invovles 2 or more experimental interventions, each wtih 2 or more variables that are studied independently

Pica develops in what disease?

- Iron deficiency anemia

Hyponatremia in heart failure is an indicator of what?

- It parallels the severeity of heart failure and is an independent predictor of adverse clinical outcomes

Lesion that chnages in color from pink to violet in a patient with risk factors for HIV?

- Kaposi's sarcoma

67 year old man. wife says has had increasing forgetfulness over the past 2 weeks. Gait has been unsteady. Over the last month, has been taking over the counter stool softeners for constipation. Smokes a pack daily. Recently started drinking home-distilled whiskey. PE shows weakness on dorsiflexion of bilateral wrists and feet. Pinprick sensation decreased over the hands and feet. Upper and lower deep tendon reflexes are 1+ What is this?

- LEAD poisoning - Distillation of alcohol through parts with lead soldering

Compare large fiber neuropathy vs small fiber neuropathy

- Large fiber: loss of pressure, propioception and balance; numbness and poor balance; diminished/absent ankle reflexes; reduced/absent vibration/light touch/propioception - Small fiber: loss of pain and temperature; burning and stabbing pain; reduced pinprick sensation; ankle reflexes possibly preserved

55 year old man. elbow pain. worse with activity. works as an airport baggage handler. With the elbow held in extension, passive flexion of the wrist reproduces the patient's pain. What is the most likely diagnosis?

- Lateral epicondylitis

Persistent ST elevation in anterior leads at least 6 weeks after MI?

- Left ventricular aneurysm

S4 most commonly associated with what?

- Left ventricular hypertrophy from prolonged hypertension

57 year old man. just returned from cruise. has pneumonia. low serum sodium. what bug is this and what is the treatment?

- Legionella - Azithromycine or levofloxacin

Patients with a new diagnosis of diabetes > 40 versus <40 in terms of treatment for mild diabetes?

- Less than 40 and lower risk ASCVD risk score (aka normal cholesterol levels, not a super elevated hemoglobin A1c): can just do lifestyle modification - Once they are over 40, they ALWAYS get put on a statin no matter what

HbA1c goal for T2DM? What do you add ACEi or ARB?

- Less than or equal to 7% - If their albumin/creatinine ratio is less than 30, it is normal and they don't need one of those drugs - Once that ratio becomes abnormal, add on ACEi or ARB to prevent diabetic nephropathy

A 37 year old man comes to the office with a year long history of fatigue and weakness. He has had a significant decrease in appetitie and interest in day to day activities due to lack of energy. also has constiaption, cold intolerance, dull constant headache, erectile dysfunction and low libido. Vital signs shows BP 98/72, normal temp, pulse 50, normal respirations. Skin is dry and pale. Delayed DTRs. Tests are small. Lab results show low sodium, low hemoglobin, normal potassium, glucose 58. What are the most likely levels of free T4, serum cortisol and aldosterone in this patient?

- Low T4, low cortisol, normal aldosterone - This is hypopituitarism - Low ACTH and cortisol: fatigue, anorexia, hypoglycemia - Hypogonadatropic hypogonadism: small testes and low libidio and erectile dysfunction - Hypothyroidism: cold intolerance, constipation, bradycardia

Algorithm for hypocalcemia?

- Low calcium - look at PTH next - PTH high - look at phosphate - PTH high, high phosphate - CKD, rhabdo - PTH high, low phosphate - vitamin D deficiency - PTH low - look at magnesium - low = hypomagnesemia - PTH low, high magnesium- hypoparathyrodism, pseudohypoparathyroidism

34 year old woman. diarrhea, fatigue and weight loss over the last year. Diarrhea occurs 2 or 3 times daily, accompanired by crampy abdominal pain. Diarrhea occasionally at night. Describes stool as very foul smelling and floating. Also compalins of diffuse bone pain. What are the levels of calcium, phosphate, and PTH?

- Low calcium, low phosphate, and high PTH - Vitamin D deficiency from malabsorption

Why does nephrotic syndrome put you at risk for accelerated atherosclerosis?

- Low plasma oncotic pressure (due to hypoalbuminemia) increases hepatic lipoprotein synthesis - This increases serum cholesterol and triglyceride levels which increases the risk for atherosclerotic disease

Low vs. normal vs. high DLCO obstructive diseases

- Low: emphysema - Normal: chronic bronchitis and asthma - Increased: asthma

Describe MEN1, MEN2A, MEN2B?

- MEN1: primary hyperparathyroidism, pituitary tumors, pancreatic tumors - MEN2A: medullary thyroid cancer, pheochromocytoma, primary hyperparathyroidism - MEN2B: medullary thyroid cancer, pheochromcotyoma, mucosal neuromas/marfanoid habitus

40 year old woman with lupus and 4 weeks of hip pain. On prednisone. Hip exam is normal, plain x ray is normal. what next?

- MRI of the hip - Avascular necrosis of the femoral head because of the prednisone treatment

Two biggest side effects of methotrexate?

- Macrocytic anemia and hepatotoxicity

Treatment for MAC infection in aIDS patients? (respiratory or disseminated)

- Macrolide antibiotics

Treatment for torsades de pointes?

- Magnesium sulfate

Walk thorugh an approach for evaluating hyperbilirubinemia.

- Mainly unconjugated (indirect): hemolysis, reduce uptake, conjugation defect ex. Gilbert - Mainly conjugated (direct) - evaluate liver enzyme pattern 1) Predominantly elevated AST and ALT: viral hepatitis, autoimmune hepatitis, toxin/drug related hepatitis, hemochromatosis, ischemic/alcoholic hepatitis 2) Normal AST, ALT and alk phos: Dubin-Johnson and Rotor syndrome 3) Predominantly elevated alk phos: cholestasis, malignancy (pancreas), cholangiocarcinoma, PBC, PSC, choledocholithiasis

Cause of tinea versicolor?

- Malassezia globosa

What is pes anserinus pain syndrome?

- Medial knee pain - Caused by abnormal gait, overuse, or trauma - Localized main at the medial tibia - x-rays usually look normal and valgus stress test does not have an effect on the pain

Most effective medications for malaria prophylaxis?

- Mefloquine until 4 weeks after return - Most countries in the world have resistance to chloroquine so we don't really use it

Dense intramembranous renal deposits that stain positive for C3?

- Membranoproliferative glomerulonephirtis - Persistent activation of the alternative complement pathway

Screening recommendations for aortic aneurysm?

- Men - Age 65-75 who have EVER smoked - One time abdominal ultrasound

65 year old. months of progressive lower back pain. irregular and hyperdense areas of bony sclerosis in the L1 and L2 vertebrae. Calcium is normal, alkaline phosphatase is high. Most likely?

- Metastatic bone lesions, most commonly prostate cancer in this gender/age - These are osteoblasti lesions, which is why the calcium is normal, unlike how it would be elevated in osteolytic disease - Multiple myeloma, for example, is lytic bone lesions so calcium woudl be high

Anion gap metabolic acidosis with epigastric pain, vomiting and blurred vision. Most likely poisoning?

- Methanol - causes optic disc hyperemia - ethylene glycol can also cause anion-gap MA, but does not damage the eyes

3 common antibiotic options to coverage anaerobes in the lungs?

- Metronidazole with amoxicillin - Amoxicillin-clavulanate - Clindamycin

Colonic biopsies that show mononuclear cell infilration of the lamina propria and thickened subepithelial band. What is this?

- Microscopic colitis - Causes water, nonbloody diarrhea, fecal urgency - Type of immune-mediated colitis

Treatment of mild vs high hypertriglyceridemia?

- Mild: 150-500: statin + lifestyle modifciations - High: >1000: fibrates, fish oil

Mild vs. moderate vs. severe hypothermia and treatment for each

- Mild: 32-25C; tachycardia, cahypnea, increased shivering; passive external rewarming (remove wet clothes, cover with blankets) - Moderate: 28-32C; bradycardia, lethargy, hypoventilation, decreased shivering; active external rewarming (warm blankets, heating pads, warm baths) - Severe: <28C; coma, cardiovascular collapse; active internal rewarming (warmed pleural or peritoneal irrigation, warmed humidified oxygen)

Hypercalcemia + metabolic alkalosis + acute kindey injury. Normal potassium. What is this?

- Milk alkali syndrome - excessive intake of calcium - look for recent heavy use of tums or like treatment for osteoporosis with things other than bisphosphonates ("mineral supplements") - Contrast with thiazide diuretic induced metabolic alkalosis with hypercalcemia - this would have hypokalemia as well

19 year old man. 2 week history of fever, fatigue and sore throat. No diarrhea or rash. No significant medical history. Sexually active with one partner, occasional protection. Temp 102.2, other vitals normal. PE shows enlarged tonsils with white exudates. Enlarged and tender cervical, axillary and inguinal lymph nodes bilaterally. What is this?

- Mononucleosis - Strep wouldn't cause symptoms for 2 weeks, would usually resolve within a week even without treatment - You could also get gonococcal pharyngitis, but wouldn't usually cause systemic symptoms such as fever and fatigue

CLL: describe, diagnostic signs and prognosis

- Most common type of leukemia in the US - almost always seen in the elderly - Extreme fatigue, B symptoms, infection or weight loss - Lymphadenopathy and splenomegaly are very often present on exam - Dramatic lymphocytosis is the hallmark - Peripheral smears: small mature lymphocytes with smudge cells - Prognosis: median survival of 10 years

3 main manifestations of progressive disseminated histoplasmosis? Treatment?

- Mucocutaneous (papules, nodules) + reticuloendothelial (lymphadenopathy, hepatosplenomegaly) + pulmonary (dyspnea and cough) - Tx: amphotericin B

Equations for NNT, AAR?

- NNT = 1/ARR - ARR = CER - EER

Most common causes of digital clubbing?

- NOT COPD OR ASTHMA - those don't usually cause clubbing - Lung malignancies, cystic fibrosis and right-to-left cardiac shunts

Treatment for pericarditis when it is idiopathic or post-viral.

- NSAIDs + colchicine

Spherocytes without central pallor + negative Coombs test vs. spherocytes without central pallor + positive Coombs test

- Negative coombs: hereditary spherocytosis - Positive coombs: autoimmune hemolytic anemia

Side effects of tacrolimus and cyclosporine?

- Nephrotoxicity - Hypertension - Neurological side effects (ex. tremor) - Impaired glucose control

Diabetic autonomic neuropathy can manifest in the GU tract how?

- Neurogenic bladder with urinary retention and distended bladder - can then develop overflow incontinence with a high post-void residual volume

Walk through the steps of evaluating a solitary pulmonary nodule.

- New lesion: look at previous chest x rays - if its been stable for 2-3 years - no further testing - Not on previous chest x-rays: do chest CT - Benign features: serial CT scans - Indeterminate or suspicious for malignancy: further investigation with biopsy or PET - Highly suspicious for malignancy: surgicla excision

How do nitrates, beta blockers, and calcium channel blockers work for angina?

- Nitrates: venodilator which increases coronary blood flow and decreases preload - Beta blockers: decrease oxygen consumption by decreasing HR - Calcium channel blockers: dilate coronary arteries which increases coronary blood flow

Do we screen for bladder cancer?

- No, it has not been shown to improve outcomes

Branching filamentous rods that are partially acid-fast?

- Nocardia

Increased thyroid hormone-binding protein level can cause what levels of TSH and T4? What things increase and decrease TBG?

- Normal TSH with high T4 - Increased TBG: estrogens, acute hepatitis - Decreased TBG: androgenic hormones, high dose glucocorticoids, hypoproteinemia, chronic liver disease

Mean median and mode relationship in negative skew, normal distribution and postivie skew.

- Normal distribution: mean = median = mode - Negative skew: mean < median < mode - Positive skew: mode < median < mean

How to treat severe hypercalcemia?

- Normal saline

Treatment of metabolic alkalosis with low potassium.

- Normal saline + potassium supplemnetation

Breath sounds, tactiel fremitus, percussion for each: - normal - consolidation - pleural effusion - pneumothorax

- Normal: bronchovesicular, normal tactile fremitus, resonant to percussion - Consolidation: increased breath sounds, increased tactile fremitus, dullness to percussion - Pleural effusion: decreased/absent breath sounds, decreased tactile fremitus, dullness to percussion - Pneumo: decreased or absent breath sounds, decreased tactile fremitus, hyperresonant to percussion

Appearance, WBC count and PMNs for normal, noninflammatory, inflammatory, and septic joint fluid analysis,. Causes of each?

- Normal: clear, <200 WBCs, <25% PMNs - Noninflammaotory (ex. OA): clear, 200-2000, 25% - Inflammatory (ex. RA, crystals): translucent or opaque, 2000-100,000, often >50% - Septic: opaque, 50,000-150,000, >80-90%

What is Morton neuroma?

- Numbness of pain between the 3rd and 4th toes - Clicking sensation when palpating space between 3rd and 4th toes

Treatment of C. diff?

- ORAL vancomycin - IV is NOT effective as it is not excreted into the colon

42 year old man with seizure disorder comes to the ED immeidatley after having seizure. Has been off seizure meds for 6 months because he hadn't had a seizure in 9 years. Appears confused and lethargic. Is found to hae an anion gap metabolic acidosis. Next best step?

- Observe and repeat the labs in 2 hours - Most likely a post-ictal lactic acidsois causing the metabolic acidosis that should resolve - If it hasn't resolved at that point, then do further incestagion for other causes - Severe acute metabolic acidosis ph <7.1, give IV bicarb

What kind of gifts can you accept from pharamceutical companies?

- Only nonmoneatry gifts that are of minimal value and that directly benefit the patient, such as unbiased educational material or drug samples should be considered

Treatment for Lyme disease?

- Oral doxycycline

Empiric first-line treatment for someone with suspected post-nasal drip?

- Oral, first-generation antihistamine

34 year old female with signs and symptoms of DVT. Also has elevated homocysteine level. Started on heparin and warfarin. What other medication should she be started on?

- Pyridoxine - Increased levels of homocysteine predispose to DVT - Homocysteine levels can usually be normalized by administration of pyridoxine and folate

60 year old man 6 month history of constant back pain, thigh pain and headahce. Deep achy pain in lumbar area that does not radiate. No significant trauma, stable. No N/V, weight loss, difficulty walking, leg numbness, or urinary symptoms. Has hypertension. Also uses hearing aid for recently acquired left sided hearing loss. Mild thoracic dextroscoliosis and a decrease in the normal lumbar lordosis. Mild anterolateral femoral bowing in the lower extremities. Most likely cause?

- Paget's disease - Osteoclast dysfunction - Skeletal deformities, bone pain, fractures, enlarging cranial bones can lead to needing a larger hat size or compression of cranial nerve 8 (hearing loss)

Most common cause of isolated asymptomatic elevated alkaline phosphatase in an elderly patient?

- Paget's disease of the bone

Treatment for aortic dissection?

- Pain control - Beta blockers - Emergent surgical repair for ascending dissection

Presentation of each: - Papillary muscle rupture - "Interventricular septum rupture - Free wall rupture - Left ventricular aneurysm

- Papillary muscle rupture: severe pulmonary edema, new holocsystolic murmur - Interventricular septum reupture: chest pain, new holosystolic murmur, biventricular failure, shock - Free wall rupture: chest pain, shock, distant heart sounds - Left ventricular aneurysm: subacute heart failure, stable angina

What hemolysis disorder more commonly gets a hypercoaguable state and portal vein thromboses?

- Paraoxysmal noctural hemoglobinuria

Complex regional pain syndrome - what xray findings?

- Patchy areas of osteopenia/demineralization

Describe nitrofurantoin lung injury?

- Patient presents with fever, dry cough, SOB, crackles, and bibasilar opacities on chest x-ray 3-9 days after taking the drug

Pemphigus vulgaris vs bullous pemphigoid?

- Pemphigus vulgaris: autoantibodies target the desmosomes (1 and 3); flaccid bullae and ulcers, separation of epiderimis by light friction; intracellular deposits of IgG; MUCOSAL invovlement - Bullous pemphigoid: autoantiboides target the dermal-epidermal junction; pruritic tense bullae; only rare mucosal involvement; IgG deposits at the basement membrane

Epigastric abdominal pain made better by food. What is this?

- Peptic ulcer disease - the food buffers the acid

Endocarditis due to Eikenella, think of what problem?

- Peridontal infection

Abscess that causes uvular deviation?

- Peritonsillar abscess

In patients with endocarditis, conduction defects (ex. heart block) shoudl raise concern for what?

- Perivavular abscess

Most common cause of B12 deficiency?

- Pernicious anemia - autoimmune condition associated wtih antibodies against intrinsic factor

What is Sheehan syndrome and describe the levels of ACTH, sodium and potassium?

- Pituitary infarction in the setting of postpartum hemorrahge - Disease of the pituitary so this is central adrenal insufficiency so deficiency in ACTH - Adrenal cortex is unaffected to aldosterone is normal and potassium is NORMAL - They don't have enough cortisol - cortiso usually inhibits antidiuretic hormone - so no inhibition and they get SIADH - so their sodium is LOW

What is the most common cause of abnormal hemostasis in patients with renal disease?

- Platelet dysfunction! - PT, PTT and platelet count are normal - VERY HIGH YIELD QUESTION

What is teh most common cancer associated wtih pleural plaques?

- Pleual plaques = asbestos exposure - BUT mesothelioma is NOT THE most common - Bronchogenic carcinoma is the most common associated with asbestos exposure

Describe the direction of tracheal shift in each of the following: pneumonia, pleural effusion, pneumothorax, atelectasis (ex. mucus plugging)

- Pneumonia: NO tracheal shift - Pleural effusion: away from effusion - Pneumothorax: away from tension pneumothorax - Atelectasis: toward atelectasis

3 main findings in tissue-invasive CMV disease?

- Pneumonitis - Hepatitis - Gastroenteritis

What test result observfed in lactose intolerance?

- Positive hydrogen breath test

Patient has normal fasting blood glucose levels but elevated A1C. what is causing this?

- Post-prandial hyperglycemia

What is amiolaride and what can it cause?

- Potassium-sparing diuretic - Can cause hyperkalemia

The width of a confidence interval correlates to its?..

- Precision - More wide = less precise

Presentation of epidermolysis bullosa?

- Presents in children and young adults with friction-induced blisters at the palms and soles - Often have chronic thickening of the skin of their feet

Antimitochondrial antibodies associated with what? Risk for what?

- Primary biliary cholangitis - Malabsorption, vitamin deficiencies, hepatocellular carcinoma,

Hypertension and undetectable plasma renin level. think of what?

- Primary hyperaldosteronism

Conn's syndrome: what is it and findings?

- Primary hyperaldosteronism - usually an adrenal adenoma or bilateral adrenal hyperplasia - High aldosterone, low renin - Hypertension, hypernatremia, hypokalemia, metabolic alkalosis

Urine osmolality < 100 suggests what? (and explain it)

- Primary polydipsia - More common in patients with psychiatric conditions, possible due to a central defect in thirst regulation - Typically develop hyponatremia and dilute urine with urine osmolality < 100 - Those with signficant urine hyponatremia can develop psychosis, seizures, confusion and lethargy

Other than colon cancer, what disease is associated wtih ulcerativce colitis?

- Primary sclerosing cholangitis

Most common cause of primary and secondary mitral regurg?

- Primary: usually mitral valve prolpase - Secondary: left ventricular papillary muscle displacement

Why does niacin cause itching?

- Prostaglandin-induced peripheral vasodilation

Explain each type of bias: - Sampling (ascertainment) bias - Nonresponse bias - Berkson bias - Prevalence (Neyman) bias - Attrition bias - Recall bias - Observer bias - Reporting bias - Surveillance (detection) bias

- Sampling (ascertainment) bias: study population differs from target population due to nonrandom selection methods - Nonresponse bias: high nonresponse rate to surveys/questionnaries - Berkson bias: disease studied using only hospital-based populations - Prevalence (Neyman) bias: exposure that happen long before disease assessment can cause study to miss patients that die early or recover - Attrition: loss to follow-up causes this type of bias - recall bias: subjectives with negative outcomes are more likely to report certain exposures that control subjects - Observer bias: *you know this* - Reporting bias: subjects over or under-report exposure history based on perceived social stigmatization - Surveillance bias; risk factor itself casues increased monitoring in exposed group relative to unexposed group, which increases probability of identifying a disease

53 year old woman. recently diagnosed with systemic sclerosis. Comes in with increasing headaches, dyspnea, malaise and nausea. She is distressed and anxious. Blood pressure 235/117, pulse 120. Lung examination shows bibasilar inspiratory crackles. Lab studies show low hemoglobin, very low platelets, high total bilirubin and high creatinine. Which red blood cell abnormality is likely to be present on a blood smear with this patient?

- Schistocytes - Marked hypertension and acute kidney injury in the setting of systemic sclerosis suggests scleroderma renal crisis (can occur in up to 20% of patients with this) - Peripheral blood smear shows a microangiopathic hemolytic anemia or DIC with fragmented RBCs (schistocytes) and thrombocytopenia

54 year old man comes to the physician complaining of morning facial puffiness and b/l leg swelling. He has recurrent pulmonary infections due to bronchiectasis and psoriasis. Blood pressure 143/92 pulse 92. Hepatomegaly, palpable kidneys and 2+ pitting edema of lower extremities. Urinalysis shows 4+ protein and normal urinary sediment. What is the most likely diagnosis?

- Secondary amyloidosis - Secondary to chornic inflammatory diseases: psoriasis, chronic infections, inflamamtory bowel disease, malignancy - Proteinuria w/nephrotic syndrome + organ enlargenment - Treat the underlying condition

Fatigue + lymphadenopathy + full body maculopapular rash + grey mucosal patches in mouth?

- Secondary syphillis

Management of ingestion of a caustic substance?

- Secure airway, breathing, circulation - Remove contaminated clothing and visible chemicals - Chest x-ray if respiratory symptoms (drooling doesn't really count, just respiratory distress, lung sounds weird, etc.) - this will identify any signs of a perforation - Endoscopy within 24 hours - assess the severeity of esophageal damage

What kind of bias does loss to follow up cause?

- Selection bias (specifically attrition bias)

When do we give pneumonia shots to adults?

- Sequently PCV13 and PPSV23 for all adults over 65 - Less than 65, PPSV23 alone for certain conditions such as heart or lung disease, diabetes, smoking, chronic liver disease

Treatment for severe, moderate, and mild hypercalcemia? (and define each)

- Severe: >14; immediate treatment is normal saline hydration, long term is bisphosphonate - Moderate: 12-14: usually no immediate treatment required unless symptomatic, use severe treatment if necessary - Mild: <12, no immediate treatment required

45 year old with nausea, fever, and upper abdominal discomfort. Medical problems include severe depression, migraine, and RA. PAtient descirbes severe tinnitus and vertigo. Admits that she overdosed on one of her medications several hours ago. Temp is 100.5, blood pressure 120/76, pulse 115, and respirations are 26. PE otherwise normal. What are the likely acid base findings? (pH, PaCO2, HCO3)

- She likely overdose on salicylates based on her findings of tinnitus, fever, tachypnea, and nausea - Remember this causes a mixed finding - respiratory alkalosis and metabolic acidosis - Low CO2 (respiratory alkalosis) - Low bicarb (metabolic acidosis) - Near normal pH as the disturbances are going in different directions

Why are women more likely to get UTIs?

- Shorter urethra - Less way for the bacteria to travel

ECG with sinus bradycardia. Telemetry with 3-6 second episodes with no sinus nodal activity. What is this?

- Sick sinus syndrome

Contraindications to anti-coagulation to treat DVT and what should you do instead?

- Significant active bleeding, recent surgery, acute hemorrhagic stroke - Place a retrievable IVC filter

Presentation and cause of GVHD?

- Skin rash, diarrhea after transplant - Activation of donor T lymphocytes - remember the GRAFT is attacking the HOST

In all patients undergoing elective surgery, what is recommended at least 4-8 weeks before surgery to decrease postoperative pulmonary complications?

- Smoking cessation

Medical treatment for TCA overdose?

- Sodium bicarbonate

Why is sodium bicarbonate used for tricyclic antidepressant overdoses?

- Sodium load will alleviate depressatn action on myocardial sodium channels

Managment of ascites in cirrhosis?

- Spironolactone wtih furosemid - Alcohol abstinence, sodium restriction - Avoid ACE inhibitors, angiotensin receptor blockers - Refractory ascites: large volume paracentesis, TIPS

Most common cause of bacterial conjunctivitis in an adult?

- Staph aureus

Most common cause of community-acquired bacterial meningitis?

- Streptococcus pneumoniae

symptoms and treatment of each: -Stress incontinence -Urgency - Mixed - Overflow

- Stress: leaking wth cough, laugh, sneez; lifestyle modificatiojns, pelvic floor exercises, pessary, pelvic floor surgery - Urgency: sudden, overwhelming, frequent need to void; bladder training, antimuscarinic drug - Mixed: features of stress and urgency; variable depending on predominant symptoms - Overflow: constant involuntary dribbling and incomplete emptying; identification and correction of underlying cause; cholinergic agonists; self-cathing

70 year old woman. found confused by her daugther. had URI a few days ago, only symptomatic treatment. No medications routinely. PE shows dry oral mucosa. Lung asucultation reveals rigth sided bronchial breath sounds and crackles. Lab results show elevated leukocytes, BUN70, creatinine 1.8. Chest radiography reveals a right lower lobe consolidation. What are the likely values of urine sodium (<20 or >20), urine sediment, and urinte white blood cells (0-1 or 5-10)

- urine sodium <20, no urine sediment (no casts), 0-1 WBC - She has a pre-renal kidney injury induced by sepsis from her pneumonia

How do you diagnose carpal tunnel syndrome?

- usually clinical diagnosis but can confirm with nerve conduction studies

Who should be screened for lung cancer?

- yearly: age 55-80 who have a 30+ pack year smoking history AND is a current smoker or quit smoking within the last 15 years

Cd4 count and symptoms for each type of diarrhea associated wtih HIV: - Crypto - Microsporidium/isosporidium - Mycobacterium avium - CMV

-Crypto: < 180, SEVERE watery dairreha, low grade fever - Microsporidium: <100, watery diarrhea, crampy abdominal pain, FEVER IS RARE - MAC: watery diarrhea, HIGH FEVER - CMV: frequent small volume diarrhea, blood in stool

For these different treatments of chronic stable angina, name the purpose/use of them and their mechanisms of action: 1) Beta blockers 2) Nondihydropyridine CCBs 3) Dihydropyridine CCBs 4) Nitrates 5) Ranolazine

1) Beta blockers: first line treatment; decrease myocardial contractility and heart rate 2) Nondihydro CCBs: alternative to beta blockers; decrease myocardial contractility and heart rate 3) Dihydro CCBs: added to beta blocker when needed; coronary artery vasodilation which decreases afterload by systemic vasodilation 4) Nitrates: long-acting added for persistent angina; decreases preload by dilation of capacitance veins 5) Ranolazine: alternative therapy for refractive angina ; decreases myocardial calcium influx

5 main medications to treat patient with coronary artery disease with?

1) Dual antiplatelet therapy 2) Beta blockers 3) ACE or ARB 4) Statins 5) Aldosterone antagonists (only in patients with LVEF <40% who have heart failure or diabetes)

Associations with each of these: 1) Focal segmental glomerulsclerosis 2_ Membranous nephropathy 3) Membranoproliferative glomerulonephritis 4) Minimal chnage disease 5) IgA nephropathy

1) FSG: African American and Hispanic ethnicitiy, obesity, HIV and heroin use 2) Membranous nephropathy: adenocarcinoma, NSAIDS, hepatitis B, SLE 3) Membranoproliferative glomerulonephritis: hepatitis B and C; lipdystrophy 4) Minimal change disease: NSAIDS; lymphoma 5) IgA nephropathy: upper respiratory tract infection

Increasing sensitivity will decrease? Increasing specificity will decrease?

1) False negatives 2) False positives

5 non-pharmacological ways to reduce blood pressure (in order from most beneficial to least)

1) weight loss 2) DASH diet 3) Exercise 4) dietary sodium decrease 5) limit alcohol intake

What are the three main dietary recommendations for people with renal calculi?

1. Increased fluid intake 2. Decreased sodium intake 3. Normal dietary calcium intake

34 year old man with premature atrial complexes found on routine ECG. no symptoms. no medical problems. smokes 1-2 packs daily and consumed 1-2 beers/day for the past 10 years. what is the next best step? A. Advise him to stop alcohol and tobacco B. Order a 24 hour Holter monitoring C. Perform a transthoracic echo D. Provide reassurance and follow-up if symptoms develop E. Start beta blocker therapy

A. Advise him to stop alcohol and tobacco - Even in asymptomatic patients, precipitating factors such as alcohol, tobacco and caffeine should be avoided

33 year old man. mild exertional SOB and "pounding:" heart over the last 5 months. Uncomfortably aware of his heartbeat while lying on his left side. Vital signs 150/45, 73. What is most likely responsible for this? A. Aortic regurigtation B. Aortic stenosis C. Mitral stenosis D. Pulmonic regurgitation E. Tricuspide stenosis

A. Aortic regurgitation - B/c of the regurg, chronic regurg increases the size of the LV - Brings the ventricular apex closer to the chest wall, causing a pounding sensation and an uncomfotably awareness of the heartbeat, particularly in the left lateral decubitus position

29 year old female. worsened generalized fatigue and lightheadedness. fatigue, nausea, diffuse abdominal pain and a 22 lb weight loss over the past 2 months. has hypothyroidism, takes levothyroixine. Works as flight attendant. Orthostatic changes present. Dry and pigmented buccal mucosa. Lung and heart sounds normal. ABdomen has mild generalized tenderness. Labs show low hemoglobin and high BUN. Most likely cause? A. Autoimmune inflammation B. Eating disorder C. Excess medication use D. Occult malignancy E. Parasitic infection F. Viral infection

A. Autoimmune inflammation - This is adrenal insufficiency - Usually caused by autoimmun response against the bilatearl adrenal cortex - Reduced aldosterone - orthostatic hypotension - Reduced feedback = high ACTH and therefore high POMC - hyperpigmentation of skin

A 34 year old woman comes to the office due to episodic mid-abdominal pain, nausea, and a watery diarrhea for the past 4-5 months.Th episodes often occur within an hour of eating, and she has also experienced increased flatus and a bloating sensation. The patient takes a PPI for GERD. She underwent a cholecystectomy 3 years ago due to symptomatic gallstones. She drinks alcohol on social occasions. The patient jogs daily and goes to the gym 3 times a wekk. Over the past several months, she has been consuming more leafy green vegetables, fruits, and low fat dairy products in an effort to lose weight. Her parents were born in China, but the patient has not traveled overseas in the past 4 years. Temp is 98.2, blood pressure 128/72, pulse 87, BMI 30. The abdomen is soft and nontender. Fecal occult blood test is negative. What is the most likely cause? A. Brush border enzyme deficiency B. Decreased bile salt absorption C. Disruption of the colonic flora D. Immune mediated enterocyte damage E. Luminal bacterial overgrowth F. Pancreatic enzyme deficiency G. Relapsing inflammation of colonic mucosa

A. Brush border enzyme deficiency - This is lactose deficiency - B) can cause diarrhea in the immediate post-operative period following cholecystectomy but resolves within a few weeks to months C) C. diff: acute onset of watery diarrhea and fever, occult blood test commonly positive D) celiac disease: foul-smelling, greasy diarrhea f) exocrine pancreatic insufficiency: typically have greasy stools and risk factors such as chronic pancreatitis, cystic fibrosis, or history of bowel or pancreatic resection g) Ulcerative colitis: abdominal pain, bloody diarrhea, fatigue and fever

52 year old woman comes in with symptoms of "pounding" sensation in her neck and lower extremity edema. Has lost weight recently. Other problems include migraine disorder, chronic diarrhea, and severe post-menopausal flushes. Echo shows retracted and immobile tricuspid valve leaflets with poor coaptation and severe tricuspid insufficiency. What is the most likely diagnosis? A. Carcinoid syndrome B. Chronic thromboembolic disease C. Infective endocarditis D. Myxomatous valve disease E. Primary pulmonary hypertension F. SLE

A. Carcinoid syndrome - Flushing, diarrhea, cardiac valvular lesions, dermatitis, diarrhea, dementia

Which of these is associated with heart failure? A. Constriction of the renal efferent arterioles B. Decreased plasma colloid pressure C. Decreased renal venous pressure D. Dilation of the renal afferent arterioles E. high sodium delivery to the distal tubule F. Increased chloride devliery to the macula densa

A. Constriction of the renal efferent arterioles - Decreases net renal blood flow and increases intraglomerular pressure to maintain GFR - There has been reduction of stroke volume and cardiac output so the kidneys have to compensate to maintain the GFR

62 year old woman. weeklong history of brief episodes of sharp chest pain that worsen with inspiration. Fatigue and stifness and pain in the hands and knees. Cardiomyoatphy. Takes hydralazine, carvedilol, forsemide, isosorbide mononitrate. Temp 100, BP and pulse normal. Inspiraotry rub is present. Moderate swelling and tenderness of the hand and knee joints bilaterally. No rash. Most likely? A. Drug adverse effects B. Hypertrophic cardiomyopathy C. Polyarticular gout D. Small-vessel vasculitis E. Thromboemoblism

A. Drug adverse effects - DRUG INDUCED LUPUS from the hydralazine

65 year old with nonischemic cardiopahty. Takes furosemide, carvediolol, lisinopril, and digoxin. Comes in with atrial fibrillation. Treated with rivaroxaban and amiodarone with improvement in symptoms. Send home on those medcations. Comes back 2 weeks later with profoud anorexia, nausea, and genrealzied weakness. What is the msot lkely cause of these symptoms? A. Drug interaction B. Erosice gastritis C. GI bleeding D. Mesenteric ischemic E. Occult carcinoma f. Pancreatitis

A. Drug interaction - Digoxin toxicity: anorexia, NV, abdominal pain, fatigue, confusion, weakness, color vision changes - Amiodarone increases the level of digoxin, so his digoxin should hace been lowered by 25-50% when they started him on amiodarone

5 year old boy brought in by mother's boyfriend for evaluation of thigh pain. This morning boy bumped into the edge of a table and now has swelling and pain in the right thigh. Family history significant for a maternal uncle who died after developing intracranial hemoorhages after a minor head injury last year. Exam shows a boy with marked ecchymoses and swelling over the right quadriceps. What is most likely to found on further history/examination of this boy? A. Episodes of joint pain with swelling B. Old, healed rib fractures C. Pinpoint red spots on extremities D. Red papules on the trunk and lips E. Sister with heavy menstrual bleeding

A. Episodes of joint pain with swelling - Hematoma after minor trauma concerning for hemophilia - Hemophilia also causes hemoarthrosis - presents with joint pain and swelling

A 24 year old man comes to the urgent care clinic due to blood in his urine. He has a 1 day history of mild midline back discomfort, associated with a single episode of red urine. The hematuria subsequently resolved and his urine is now a normal yellow color. The patient also has had a sore throat for the past 4 days, which he took 3 tabs of tylenol fore. He has had no fever, abdominal pain, or burning during urination. He has smoked 1 pack of cigarettes daily for 5 years. Physical exam is unremarkable. Urinalysis shows 3+ protein and 50-75 RBC/hpf. Which of the following is the most likely source of this patient's urinary complaints? A. Glomerulus B. prostate gland C. Renal interstitium D. Renal papilla E. Renal tubule F. Urinary bladder epithelium

A. Glomerulus - This is glomerular hematuria - red blood cells + protein - Post-streptococcal glomerulonephritis, IGA nephorpathy, basement membrane disorder, etc.

A 64 year old man comes to the physician due to shortness of breath and abdominal distension. He was treated for Hodgkin lymphoma with radiation and chemotherapy 18 years ago and was told that he was cured. The patient drinks alcohol on a regular basis. His temp is 36.7, blood pressure 120/76, pulse is 92, and respirations are 20. Neck examination shows JVP 9 cm above the sternal angle. The abdomen is distended with a positive fluid wave. The liver edge is palpated 5 cm below the right costal edge. There is bilateral lower extremity pitting edema. Initial lab results show normal creatinine, normal albumin, normal total bilirubin, and normal prothrombin time. What is the most likely cause? A. Inelastic pericarditis B. Portal vein compression C. Portal vein thrombosis D. Thoracic duct obstruction E. Urinary protein loss F. venous valve incompetence

A. Inelastic pericarditis - This is constrictive pericarditis - late complication of radiation therapy

60 year old man. fatigue and muscle weakness in extremities. Has lost 15 in the last 3 months. No other medical conditions. Smokes 2 packs of cigarettes daily. Muscle stregnth 5/5 in proximal muscle groups symetrically. Reflexes normal b/l. Erythematous to violaceous papules invovling the dorsum of his fingers. Chest x-ray revecals an ill-defined mass in the right lower lobe. Muscle weakness due to a lesion invovling what? A. Muscle fibers B. Peripheral nerves C. Postsynpatic membran e D. Presynaptic membrane E. Spinal cord

A. Muscle fibers - This is dermatomyositis - REMEMBER, this can be a paraneoplastic syndrome

56 year old woman. epigastric abdominal pain for a year. worsens after eating and crampy intermittent and nonradiating. Bloating and nausea. no weight loss, vomiting, constaiption, diarrhea, melena or hemotochezia. Hypothyoridism and hypertension.No alcohol or tobacco. Abdomen mildly tender to palpation in the epigastrium without rebound, guarding or distension. Labs show hemoglboin 10.2, MCV 105. What is the most likely? A. Autoimmune atrophic gastritis B. Functional dyspepsia C. Gastroparesis D. H. pylori peptic ulcer D. Irritable bowel syndrom e E. Small intestinal bacterial overgrowth

A. autoimmune atrophic gastritis - She is anemia - it can't just be functional - Autoimmune disorder, often occurs in conjunction wtih other autoimmund conditions - Antibodies against parietal cells causing B12 deficiency

Which one of these is seen in septic shock? A. bounding pulses B. cold extremities C. distended neck veins D. pulsus paradoxus E. third heart sound

A. bounding pulses B can be seen in the more chronic phase of septic shock after the bounding pulses go away C. would be congestive heart failure D. would be cardiac tamponade E. congestive heart failure, volume overload

43 year old man hospitalized for acute pancreatitis. Day 3 of admission, respiraotry distress. Transferred to ICU and intubated. Initial ventilator settings are: fraction of inspired oxygen 80%, respiratory rate 14, tidal volume 380 mL, PEEP 7 cm H2O. Ten minutes later, the patient's blood pressure is 110/70 and heart rate is 90. Ph is 7.42, CO2 is 37 and O2 is 105. What is the next best step? A. Decrease the fraction of inspired oxygen B. Decrease the PEEP C. Decrease the respiratory rate D. Decrease the tidal volume E. Increase the respiratory rate

A. decrease the fraction of inspired oxygen - Goal with this is to maintain PaO2 at 55-80 - His blood gas shows a normal CO2 and more than adquate oxygenation - Because prolonged high FIO2 can cause oxygen toxicity - The goal is to decrease down to <60% to prevent this from happening as soon as you have them stabalized, which he is

45 year old woman. 1 day of fever, chills, nausea and vomiting. Progressively worsening R upper quadrant pain for 2 days. History of depression and suicide attempt as teenager. No medications, or illicit drugs. Temp is 103.1, blood pressure 90/48, pulse 98. BMI 36. Appears ill and confused. Skin and sclera are icteric. Cardio exam normal. Marked tenderness/guarding at RUQ. Lab results show total bilirubin of 7.2, direct bilirubin of 5.4, alk phos 714, AST 75, ALT 60. Lipase 81 (normal). What is most likely to be found? a. dilation of the intrahepatic and common bile ducts B. inflammed and edematous enlargement of teh pancreas C. markedly elevated serum acetaminphen levels D. short, annular strictures alternating with normal bile duct E. thickening of the gallbaldder wall and pericholecystic fluid

A. dilation of the intrahepatic and common bile ducts - Acute cholangitis - High direct bilirubin with only mildly elvated AST/ALT are more likely to be cholecystitis/cholangitis - If total bilirubin was normal/close to normal - acute cholecystitis (option E) - high bilirubin shows us it is a biliary obstruction

35 year old man. year of weakness, fatigue and weight loss. Has also experienced reduced appetitie and intermittent diarrhea. No significant medical history. No tobacco/alcohol/drugs. Family history sister has hypothyroidism. Vital signs are normal. He is in no acute distress. Physical exam normal. Labs show low hemoglobin, low WBCs, low serum sodium, low serum potassium, normal 8 am cortisol and normal TSH. What is the next most appropriate in managemnet? A. 24 hour urine free cortisol B. ACTH stimulation test C. insulin-induced hypoglycemia test D. intravenous hydrocortisone E. Low-dose overnight dexamethasone suppression test

B. ACTH stimulation test - This is primary adrenal insufficiency (Addison's) -

A 57 year old man comes to the physician for 2 episodes of blood in the urine. He also complains of faituge and fever for the past 4 weeks. He has no other medical problems and takes no medications. The patient has a 50 pack year smoking history but not use alcohol or ilicit drugs. His father died from a blood disorder but the patient is unsure of the specific name. Vital sigsn are normal. Examination shows a left-sided varicocele that fails to empty when the patient is recumbent. The remainder of the exam is normal. Lab results show high hemoglobin, normal WBCs, high platelets, and blood in his urine. Which of the following is the most appropriate diagnostic proceudre? A. Chest x-ray B. Abdominal CT scan C. Urine cytology D. Serum alpha=fetoprotein levels E. Ultrasound of the testicles F. bone marrow biopsy

B. Abdominal CT scan - This is metastatic renal cell carcinoma - Most patients are asymptomatic until teh disease is advanced - Hematuria signifies the tumor invading the collecting system - The varicocele is the tumor obstructing the gonadal vein

50 year old man with blurry vision over the last 6 months. Difficulty focusing on small print while working at his computer and often develops a mild headache. History of allergic rhinitis. BMI 25. Blood pressure 130/76 pulse 78. On funduscopy, retinal venules are narrowed at the intersection points with arterioles and appear to bulge on either side. What next step? A. 24 hour ambulatory ECG monitoring B. Ambulatory blood pressure monitoring C. Coronary angiography D. Exercise stress testing E. MRI of the brain

B. Ambulatory blood pressure monitoring - Masked hypertension - Normal when he gets BP taken, but actually high - Look at the retinal findings - hypertensive retinopathy

65 year old man with known ischemic cardiomyopathy. 2 weeks progressive SOB and dry cough. Had an MI 5 years ago, severe left ventricular systolic dysfunction with an EF of 20%. He has an ICD. Six months ago he was hospitalized with recurrent ACID shocks due to v. tach - sucessfully treated wtih antiarrhythmic therapy, still currently taking. Jugular veins flat, bilateral inspiratory crackles are heard. No cardiac murmurs. No significan periperhal edema. Chest x-ray reveals b/l lung infiltrates involving primarily the middle lung fields. What is the most likely cause? A. Advanced heart failure B. Antiarrhythmic toxicity C. Diuretic toxicity D. Tricuspid valve insufficiency E. Viral pneumonia

B. Antiarrhythmic toxicity - Amiodarone is often used for managemnet of V tach in patients with coronary artery disease and ischemic cardiomyopathy - Amiodarone can cause an interstitial pneumonitis, usually witnin MONTHS, of starting therapy - Presents with progressive dyspnea, nonproductive cough and new reticular or ground-glass opacities on chest radiograph

54 year old man comes in with tearing chest pain. Acute dissection of the ascending aorta is confirmed. While being prepped for surgery, he develops increasing SOB, and refuses to lie flat. Chest auscultation revelas bibasilar crackles that were not present on initial evaluation. What is the most likely cause of this patient's SOB? A. ARDS B. Aortic valve insufficiency C. Cerebral hypoperfusion D. PEricardial tamponade E. Pulmonary arterial hypertension

B. Aortic valve insufficiency - Retrograde extension of the tear can invovle the aortic valve and cause acute aortic regurgitation, which presents with these symptoms - Tamponade is another possible complication of aortic dissection, but DOES NOT have pulmonary edema

29 year old man comes to clinic 2 weeks after ED visit for epistaxis requiring anterior nasal packing. In the ED his BP was 170/110. BP is 180/112 today. Occasional headaches and fatigue. No chest pain, palpitations or syncope. No PMH. No mumurs or additional sounds. No periumbilical bruits. ECG normal sinus rhythm, high-voltage QRS complexes, downsloping ST segment depression, and T waves inversion in leads V5 and V6. Labs normal. Next best step? A. Ambulatory blood pressure monitoring B. Bilateral arm and leg blood pressure measurements C. Cardiac auscultation in squatting and standing positions D. Carotid sinus massage E. Exercise stress testing

B. B/l arm and leg blood pressure measurements - Coarctation of the aorta - The ECG findings are called basically a "stress" pattern - suggestive of LV hypertrophy - He is very young to have primary hypertension

62 year old man comes to the office due to an abnormal chest xray. 2 weeks ago, enlarged mediastinal lymph nodes found incidentally. No fever, cough, or dyspnea but has had nonpainful nodules in his axillae for the past several months. Has hypertension. Lifelong nonsmoker. PE reveals clear lungs and normal heart sounds. Bilateral axillary and inguinal lymph nodes are enalrged and nontender. Chest x-ray from a year ago shows mediastinal lymphdenopathy. Most likely cause? A. Antinuclear antibody formation B. BCL-2 gene overexpression C. Disseminated fungal infection D. Drug hypersensitivity reaction E. Metastasis of a lung neoplasm

B. BCL-2 overexpression - Long-standing lymphadenopathy raises concern for follicular lymphoma - Most patients present with months to years of painless peripheral lymphdenopathy

A 37 year old man is brought to the ED. Police found him running into traffic, yelling obscenities at drivers, slamming his body against passing cars, and trying to break their windshields. A witness described him as having "crazy rage." In the ED, the patient remains combative with police and 4 officers are needed to restrain him. He is disoriented, continues to yell, and appears to be responding to internal stimuli. Temp is 99, blood pressure is 190/96, pulse is 142, and respirations are 20. Examination shows no abnormalities except for severe agitation and hyperreflexia. Serum lab results are normal and urine toxicology is positive for tetrahydrocannabinol. He is admitted to the medical service where further medical and neurological tests, including a lumbar puncture, are negative. The patient remains psychotic for 2 days, but his symptoms then begin to subside and eventually disappear with occasional use of benzodiazepines and supportive care. Which of the following is the most likely explanation for this patient's presentation? A. Acute schizophrenia exacerbation B. Bath salts intoxication C. Cannabis intoxication D. Cocaine intoxication E. Ecstacy intoxication F. PCP intoxication

B. Bath salts intoxication - PCP intoxication is of a shorter duration than bath salts, would probably have nystagmus, and usually shows up on drug screens

34 year old previously healthy woman. Itchy eyes for several months. Reports frequent gritty sensations in her eyes and crusting of her eyelashse in the morning. No eye pain, photophobia, or vision disturbances. Seborrhea present at the scalp and eyebrows, but the skin of the face is normal. Nasal and oropharyngeal mucosa are normal. What is the most likely? A. Allergic conjunctivitis B. Blepharitis C. Dry eye disease D. Hordeolum E. Preseptal cellulitis

B. Blepharitis - Inflammation at the eyelid margin - Common contributors include seborrheic demaritis (like this person has), rosacea, allergic disorders, bacterial infection, and viral infection

54 year old woman who has chronic idipathic pancreatitis. Comes in with worsening epigastric pain for 2 months, and weight loss of 15 lb over the last 3 months. No chnage in stool or urine. No tobacco/alcohol/drug use. No jaundice. Abdominla exam shows mild epigastric tenderness on deep palpation. What is the next msot appropriate step? A. Cancer associated antigen testing B. CT scan of the abdomen with contrast C. Endoscopic retrograde cholangiopancreatography D. Intensification of pancreatic enzyme replacment therapy E. Plain abdominal radiography F. Reassurance and opioid therapy G. Secretin pancreatic function testing

B. CT scan of the abdomen with contrast - weight loss + gnawing abdominal pain in the setting of known chronic pancreatitis = PANCREATIC CANCER - Cancers in the head of the pancreas usually have jaundice - you would do abdominal ultrasound - Cancers in the body and daily DO NOT have jaundice - do CT scan

A 52 year old man comes to the office due to decreased libido and inability to achieve and maintain erections for the past several months. He has also had fatigue, anorexia and a 11 lb weight loss over the same duration. the patient is married and has one child. he drinks alcohol and does not use tobacco or illicit drugs. Blood pressure is 110/70 and pulse is 82. Physical examination shows bilateral gynecomastia and firm and small testes. Lab testing shows normal TSH with decreased levels of total T3 and T4. What is the most likely? A. Adrenal insufficiency b. Chronic liver disease C. Exogenous thyroxine intake D. Hashimoto thyroiditis E. Penile vascular occlusion F. testosterone insensitivity

B. Chronic liver disease - Cirrhosis can cause hypogonadism (small testes) - Liver disease also causes increased conversion of androgens into estradiol causing the gynecomastia - Finally, the liver produces binding proteins for thyroid hormones - so cirrhosis lowers the production of these hormones causing decreased total levels of T3 and T4 but normal free T3/T4 levels and normal TSH levels

25 year old woman transient vision loss in right eye. Six months ago told blood pressure was high. No PMH otherwise. No medications. Grandmother had a stroke at 50. Blood pressure 164/103. BMI 26. Bruit below right mandibular angle. Lab results show normal potassium, high plasma renin activity. What is the next best step in management of this patient? A. Adrenal venous sampling B. Computed tomography angiography of the abdomen C. Dexamethasone suppression test D. Erythrocyte sedimentation rate E. Plasma fractionated metanephrines

B. Computed tomography angiography of the abdomen - Fibromuscular dysplasia - onset of hypertension before the age of 35 is suspicious here - Non-inflammatory and nonatherosclerotic condition caused by abnormal cell development in the arterial wall - can occur in any vessel but classically affects renal, carotid and vertebral arteries - REsistant hypertension, signs of brain ischemia (amaurosis fugax, Horner's, TIA, stroke), nonspecific symptoms (headache, pulsatile tinnitus, dizziness) from carotid or vertebral artery involvement - Test with computed tomography angiography, duplex ultrasound

68 year old man. diarrhea for the past 2 months. stools are postprandial, watery, foul-smelling and difficult to flush. Intermittent, epigastric, postprandial abdominal pain. Unintentionally lost 15.4 lb. Med history significant for a prolonged hosptialization 3 years ago for acute necrotizing pancreatitis. Vital signs normal. PE shows aniceteric sclera, moist mucous membranes, and normal heart and lung sounds. Abdomen is midly tender to palpation in the epigastrium. Most likely to be present in this patient? A. Abnormal lactose breath test B. Decreasd fecal elastase C. Increased fecal calprotectin D. Increased fecal leukocytes E. Tissue transglutaminase antibodies

B. Decreased fecal elastase - History of necrotizing pancreatitis + weight loss/abdominal pain/steatorrhea suggests chronic pancreatitis - Elastase is a pancreatic enzyme that is low with injury to the pancreas

53 year old man. worsening weakness and exertional dypsnea for 2 days. Upper URI 3 weeks ago. No chest pain or palpitations. Blood pressure 87/60, pulse 109. Oxygen sat 95% RA. BMI of 23. JVP 11. Lungs clear. Heart sounds muffled. What is the most likely cause? A. Decreased cardiac contractility B. Decreased left ventricular preload C. Increased right ventricular compliance D. Left ventricular outflow obstruction E. Pulmonary hypertension

B. Decreased left ventricular preload - BECK'S TRIAD: HYPOTENSION + MUFFLED HEART SOUNDS + DISTENDED NECK VEINS - CARDIAC TAMPONADE - reduced left ventricular preload, decreased stroke volume, decreased cardiac output

A 45 year old woman, reflux, nause aand vomiting for the past 3 months. Daily heartburn, regurgitation and a sour taste in her mouth. Also has severe nocturnal reflux and has been sleeping in a recumbent position. Postprandial, nonbloody, nonbilious emesis. Underwent a Roux-en-Y gastric bypass 4 months ago. Vital signs normal. There is mild epigastric tenderness without guarding or rebound tenderness. What is the next best step? A. Avoidance of simple carbs B. EGD C. Gastric emptying scan D. Right upper quadrant ultrasound E. Trial of proton pump inhibitor

B. EGD - Most likely has stomal (anastomatoic) stenosis - diagnosis requires visulaization of the anastamosis via EGD during which balloon dilation can be done to open the stenosis

32 year old man brought to ED with excruciating, sudden-onset chest pain and neck pain. He is 6 ft 5. The hand joints show significant extensibility. Pectus carinatum deformity is present. Which of the following additional findings is likely in this patient? A. Audible S3 B. Early diastolic murmur C. Fixed splitting of S2 D. Opening snap E. Pericardial friction rub F. Pulsus parvus et tardus

B. Early diastolic murmur - Marfan syndrome - probably acute aortic dissection - Aortic regurgitation - early diastolic murmur

34 year old, G1P0, comes in at 38 weeks due to increasing SOB. Pregnancy is causing her too much discomfort and she requests devliery. Walking severeal steps she needs to stop and catch her breath, can't find a comfortable sleeping position. Uncomplicated pregnancy. BP 140/85, pulse 102 and regular. Third heart sound is present, grade II/VI holosystolic murmur at apex. Pitting edema present b/l. Urine shows trace protein. ECG shows normal sinus tach. What next? A. Chest x-ray B. Echo C. Immediate delivery D. Mag sulfate E. reassurance and routine care

B. Echo - This is peripartum cardiomyopathy - onset of heart failure during the last month of pregnancy or within 5 months following delivery - Absence of other causes of heart failure - Absence of heart disease prior to final month of pregnancy

A 62 year old man comes to the office due to difficulty swallowing both solids and liquids. The patient's symptoms have progressively worsened over the last 3 months. He also has occasional regurgitation of undigested food and a nighttime cough that disturbs his sleep. He has never had similar symptoms before. The patient has lost 10 lb during this period but has no other medical problems and takes no medications. He has a 20 pack year smoking history but quit 10 years ago. he drinks wine on the weekends. Vital signs are normal, and his physical examination is unremarkable. chest x-ray reveals a widened mediastinum and barium studies show a dilated esophagus with smooth tapering of the distal esophagus. Which of the following is the best next step in management of this patient? A. CT scan of the chest B. Endoscopic evaluation C. esophageal ph monitoring D. Laparoscopic myotomy E. Reassurance and observation F. Trial of proton pump inhibitor

B. Endoscopic evaluation - Narrowing = bird's beak - Suggests achlasia or pseudoachalsia due to esophageal cancer - Most likely cancer in this case based on his risk factors

77 year old man. 2 weeks of feveres and generalized weakness. Had a hospitalization for pyelonephritis requiring IV antibioitics 3 months ago, episode of rheumatic fever as a child, and Hodgkin's lymphoma treated 15 years ago. Recently underwent cystoscopy for evaluation of persistent dysuria. temp of 100. New holosystolic murmur at the apex and tender eyrthematous lesions affecting several fingertips. What is the most likely cause? A. Coxiella burnetti B. enterococci C. Staph epidermidis D. Strep pyogenes E. Viridans strep

B. Enterococci - RECENT urinary procedure and infection - Viridans strep is very common but associated with dental procedures/URI

25 year old man comes to ED with difficulty swallowing. Has had a piece of steak stuck substernally for 12 hours. Has failed to dislodge it despite inducing himself to vomit. He has never had trouble swallowing liquids or solids previously, althoug he has had freqeunt heartburn for the past few years and has tried a PPI without improvement. He had bronchial asthma since childhood. Vitals signs are normal. When presented with a glass of water, he vomits the water seconds after swallowing, but the piece of steak remains stuck. What is the most likely explanation for this presentation? A. Disordered neuromuscular transmission in the oropharynx B. Eosinophilic mucosal infiltration o the esophagus C. Esophageal myenteric plexus degeneration D. fibrosis and atrophy of esophageal smooth muscle E. Ulcerated circumferential infiltration of the lower esophagus with malignant cells

B. Eosinophilic mucosal infiltration of the esophagus - This is eosinophilic esophagitis - Dysphagia, epigastric pain, reflux, food impaction, associated atopy - Look for history of seasonal allergies, eczema and asthma

17 year old boy has fatigue. 6 weeks. Generalized itcvhing but no rash. No nasal congestion, sore throat, cough, dyspnea, abdominal pain or diarrhea. No PMH, no meds. Temp is 100.4, blood pressure 118/66, pulse 86. PE shows no scleral icterus or conjunctival pallor. Severeal enlarged, nontender, and firm cervical and supraclavicular lymph nodes. Mucosa and tonsils are normal. Lungs are clear, heart sounds normal. Abdomen is soft and nontender with no organomegaly. No extremity edema or skinr ash. What is the most important to establish a diagnosis for this patient? A. ANA antibody B. Excisional lymph node biopsy C. Heterophile antibody testing D. Rapid plasma reagin E. serum erythropoietin level testing F. Tuberculin skin testing

B. Excisional lymph node biopsy - This is Hodgkin lymphoma - Painless lymphadenopathy, B symptoms, pruritus

A 37 year old woman comes to the office due worsening leg swelling for the past 2 months. SHe has also felt excessively tired and has had achy pain in her hands during this time. The patient reports occasional respiratory illness and minor aches and pains. SHe has no chronic medical conditions and takes no medications. She occasionally drinks 1 or 2 glasses of wine, but no tobacco or drugs. Tep 99 BP 156/92 pulse 86. PE shows normal JVP, dullness and decreased breath sounds at the left lung base, normal heart sounds and no abdominal organomegaly. 3+ b/l LE pitting edema. Bilateral finger joints and wrists are mildly swollen and tender to palpation. Lab studies reveal a hemoglobin of 9.1, platelet count 102,000 serum creatinine 2.1 A peripheral blood smear is normal. Urinalysis is positive for proteinuria and heamturia. What is most likely? A. Filtration of excessive free light chains B. Formation of anti-DNA immune complexes C. Infection from nephritogenic strain of Strep D. Ischemia and infarction of the renal medulla E. Microthrombotic occlusion of renal vasculature

B. Formation of anti-DNA immune complexes - This is SLE - Glomerulonephritis, fatigue, inflammatory arthritis, anemia, thrombocytopenia

65 year old woman with 2 month history of fatigue and weight gain. Patient has RA, peptic ulcer disease and hypertension. PE shows generalized edema, liver is 5 cm below costal margin. UA shows 4+ protein. U/S of kidneys shows b/l enlargement. Renal biopsy would show what? A. glomerular crescent formation on light microscopy B. Glomerular deposits seen after special staining C. Hyalinosis of walls of afferent and efferent arterioles D. Linear glomerular deposits seen on immunofluorescence E. Normal light microscopy findings

B. Glomerular deposits seen after special staining - Most likely amyloidosis - RA predisposes to this

27 year, G1 P0 comes in at 9 weeks for routine care. Feeling well. No pmh. Mother and sister have anenmia. Vegetarian diet. Menses previusly occured monthly with 5-6 days of bleeding. Vital signs and PE normal. Lab studies show low hemoglobin, MCV 75, normal ferritin, normal iron, and normal Iron bidning capacity. Ultrasound shows normal 9 week preganncy. What is the next best step? A. Folate level B. Hemoglobin electrophoresis C. Lead level D. Reassurance and routine prenatal care E. TSH level F. Vitamin B12 level

B. Hemoglobin level - This is NOT iron deficiency based on the lab values, but she still has a microcytic anemia for some reason - This is NOT normal - Need to rule out something like a thalassemia

A 43 year old man is being evaluated for erectile dysfunction, which has been progressive over the past yera. HE has not other symptoms except some fatigue that he attributes to his intense work schedule. The patient drinks 1-2 beers every day after work and has a 20-pack year smoking history. He does not use drugs or take medications. Vital signs normal. BMI 29. Skin exam shows brownish skin pigmentation, most pronounced over the face and both arms. The abdomen is soft and nontender. Cardiopulm exam shows no abnormalities and there is no peripheral edema. Lab results show: fasting glucose 130, AST 78 and ALT 80. If untreated, this patient is at increased risk for developing which of the following complications? A. Gastric adenocarcinoma B. Hepatocellular carcinoma C. Hypotensive crisis D. Nephrotic syndrome E. Pancreatic cancer F. Skin cancer

B. Hepatocellular carcinoma - This is hereditary hemochromatosis - Hyperpigmentation + elevated glucose - BRONZE DIABETES - Elevated hepatic enzymes, secondary hypogonadism and hypothyroidism - Arthralgia, arthropathy and chondrocalcinosis - Restrictive or dilated cardiomyopathy and conduction abnormalities

36 year old man with 2 days of fever, headache and vomiting. Found to have meningococcal meningitis. Patient is informed. He refuses hospital admission and insists on being treated at home. He is alert and oriented and appears to understand the risk he poses to himself and others by refusing hospitalization. What do you do next? A. Consult hospital ethics commitee B. Hospitalize and isolate the patient against his wishes C. Inform the patients' wife and ask her to convince him to agree to stay D. Obtain a court oder to proceed with treatment E. Respect the patient's decision and arrange for home antibitoics

B. Hospitalize and isolate against his wishes - Patients have the right to refuse treatment except when doing so poses a serious threat to public health - Rare situation where you override patient autonomy

54 year old man. increasing left flank pain, nausea, and hematuria for 5 days. Pain is dull, constant and nonradiating. Yesterday, noticed swelling of left testis. Diagnosed with idiopathic membranous nephropathy 6 months ago. Vital signs normal. Examination reveals left flank tenderness. Dilated and tortuous veins are noted in the left scrotum. Bilateral 1+ pitting edema present. What underlies this acute presentation? A. Decreased plasma oncotic pressure B. Hypercoagulability C. Increased urine calcium excretion D. Malignancy E. Urinary tract infection

B. Hypercoagulability - He has acute renal vein thrombosis - the membranous nephorpathy puts him at risk for this by making him hypercoagulable

A 70 year old man comes to the emergency department due to severe left ear pain. The pain has progressively worsened over the past 2 weeks. It is especially severe at night and is exacerbated by chewing, so the patient has difficulty sleeping eating. He has been unable to wear his hearing aid on the left side, so he is unsure whether his hearing has worsened. Medical history is significant for hypertension, coronary artery disease, and type 2 diabetes mellitus complicated by peripheral neuropathy. Temperature is 100 F, blood pressure is 140/90, and pulse is 98. On examination, the left external auditory canal is mildly edematous with purulent discharge and granulation tissue on the canal floor. The tympanic membrane is clear. Aside from hearing loss, cranial nerve examination is normal. Complete blood count is normal, and the ESR is 89. Which of the following is the best initial treatment for this patient? A. IV ampicillin/sulbactam B. IV ciprofloxacin C. Surgical excision D. Topical corticosteroids E. Topical neomycin

B. IV ciprofloxacin - This is necrotizing (malignant) otitis externa - Risk factors include age >60, diabetes, and aural irrigation - Pseudomonas - Severe, unremitting ear pain (worse at night and with chewing) - Treat with IV ciprofloxacin first, then surgical debridement if necessary

25 year old woman 3 month history of weight loss, irritabilty, insomnia and palpitations. No PMH. No medications. Blood pressure 155/70, pulse 110. PE shows lid retraction, dry skin, and fine tremor of the hands. What is the most likely cause of this patient's hypertension? A. Elevated serum calcium levels B. Increased myocardial contractility C. Increased peripheral vascular resistance D. Increased synthesis of catecholamines E. Nocturnal hypoventilation

B. Increased myocardial contractility - This is thyrotoxicosis - Can produce a number of cardiovascular complications - Causes hyperdynamic cardiovascular state characterized by tachycardia, systolic hypertension, and widened pulse pressure - Increased contractility, increased caridac output - Decreased SVR - Increased myocardial oxygen demand

37 year old man comes in after coughing up a small amount of bright red blood. 3 days of fever, pleuritic chest pain and cough productive of thick, brown sputum. STarted on oral antibiotics for these symptoms 2 days ago, but no improvement. Had a stem cell transplant 6 weeks ago, complicated by GVHD. Temp is 102.2, pulse 112. Pulm exam shows right sided crackles. Chest x-ray reveals dense right upper lobe infiltrate. CT scan reveals several nodular lesions with surroudning ground-glass opacities in the right upper lobe. Sputum gram stain shows inflammatory cells but no organism. What is the most likely infection? A. CMV B. Invasive aspergillosis C. invasive cadidiasis D. PJP E. Pulmonary tuberculosis

B. Invasive aspergillosis - nodules with surroudning ground glass opacities = "halo sign"

66 year old man. follow up of hypertension. two months ago, BP was 165/95, patient has since had 2 follow-up visits, both documenting high blood pressure. Hasn't had any symptoms. Has T2DM and hyperlipidemia. Underwent a stent placement for peripheral vascular disease 2 years ago. Former cigarette smoker, 25 pack year history. Currently BP is 162/93, pulse is 73 and regular. BMI 31. Serum creatinine is 0.8. CT angiography reveals 80% atheroscelrotic narrowing of the right renal artery. In addition to antihyperlipidemic therapy, which of the following is the best next step in management of this patient? A. Doxazosin B. Lisinorpil C. Loop diuretic D. Renal artery stenting E. Surgical revasculatization

B. Lisinopril - Renal artery stenosis - ACEi/ARBs are first-line therapy

27 year old with recent dental procedure. SIgns/symptoms of endocarditis. What do you do next? A. Obtain anti-streptolysin O titers B. Obtain bacterial blood cutlures C. Perform TEE D. Perform TTE E. Provide symptomatic therapy with close outpatient follow up F. Start antibiotics immediately

B. Obtain bacterial blood cultures - Recommended that a minimum of 3 blood cultures be obtained from separate venipuncture sites over a specified perio d prior to initiating antibiotic therapy - Improves microbiological yield and ensures the causative organisms is identified

56 year old man episode of transient right eye blindness that lasted an hour. No weakenss in extremities or speech changes. Last saw a phsyciain 3 years ago. Smoked a pack daily for 30 years. High BP, otherwise normal vital signs. BMI is 32. No carotid bruit, facial plethora is present. Cardiopulmonary exam is normal. Splenomegaly present. Lab results show a hemoglobin of 20, platelets of 545,000 and leukocytes of 13,800. What is the best therapy for this patient? A. Anticoagulation B. Phlebotomy C. Plasma exchange D. Prednisone E. Splenectomy

B. Phlebotomy - This is polycythymia vera - Hypertension, erythomelalgia (burning cyanosis in hands and feet), transient visual disturbance, aquagenic pruritis - Facial plethora, splenomegaly - Treat with phlebotomy

65 year old male. fatigue, poor appettiie, and lower leg swelling. PMH significant for hypertension, T2DM, chronic cough, chest infections and lower extremity peripheral artery disease. 40 pack year smoking history. PE shows barrel shaped chest, bilateral scattered wheezing. Patient's abdomen is distended, liver edge 4 cm below right costal margin. 2+ pitting edema and dilated tortuous superficial veins over both lower extremities. Manual pressure of the abdoemn cause persistent distensions of the jugular veins. Abnormatliy of which is the cause of the patient's edema? A. Portal venous resistance B. Pulmonary artery systolic pressure C. Pulmonary capillary wedge pressure D. Serum albumin level E. Urinary protein excretion

B. Pulmolnary artery systolic pressure - COPD with cor pulmonale - Impaired function of the right ventricel caused by pulmonary hypertension that occurs due to underlying diseases of the lung, pulmonary hypertension, or OSA

21 year old woman evaluated for syncope. Has passed out while standing in a crowded church. Typically preceeded by warmth, lightheadedness, diaphoresis and nausea. No major medical history. No medications, no drugs. Orthostatic vital signs negative. Pulse 72 and regular. ECG shows normal sinus rhythm with no signficant problems. Long term ECG monitoring s likely to show what imemdiately preceding and during the episode? A. atrial fibrillation B. Bradycardia and sinus arrest C. sinus tachycardia D. third degreee AV block E. ventricular tachycardia

B. bradycardia and sinus arrest - VAsovagal syncope - Most commonly, abrupt parasympathetic activation leads to a cardioinhibitory response that manifests as bradycardia with sinus arrest

Which one of these is a complication of Paget's disease? A. Acute renal failure B. Hearing loss C. Liver cirrhosis D. Pulmonary hemorrhage E. Renal cell carcinoma F. Subarachnoid hemorrhage

B. hearing loss

45 year old male. several months of easy fatigability and exertional dyspnea. subtotal gastrectomy for nonhealing gastric ulcer. shiny tongue and pale palmar creases. what pathologic mechanisms? A. immune related red blood cell injury B. impaired DNA synthesis C. impaired globin chain synthesis D. impaired glutathion synthesis E. mechanical red blood cell injury F. red blood cell membrane instability

B. impaired DNA synthesis - B12 deficiency - B12 is needed for DNA synthesis

A 31 year old Caucasian male with joint pains. Right knee pain, right heel pain and low back pain. Recently treated for urethral discharge. No trauma or drug use. Afebrile. Knee is swollen warm and tender. Mouth ulcers present. Synovial fluid analysis shows high WBCs, with many PMNs but negative Gram stain. Most appropriate intial therapy? A. antihistamines B. nsaids C. antibiotics D. colchicine E. allopurinol F. chondroitin sulfate

B. nsaids - This is reactive arthritis NOT gonococcal infection - Triad of nongonoccal urethritis + asymmetric oligoarthritis + conjunctivitis

68 year old man comes to ED with palpitations and mild dizziness and sweating. STarted 30 minutes ago. No chest pain or SOB. Hypertension and MI 6 months ago. Ex smoker with 30 pack year history. Blood pressure 102/65, pulse 160 and reulgar. Oxgeyn sat 2% on 2 L nasal cannula. On PE, prominent, high-amplitude jugular venous pulsations are seen intermittently are irregular intervals. ECG reveals a regular, wide-complex tachycardia. Which of the following best explains these physical findings? A. Aortic regurgitation B. Atrial fibrillation C. Atrioventricualr dissociation D. Pericardial effusion E. Tricuspid regurgitation

C. Atrioventricualr dissociation

which one of these would be increased with a GI bleed? A. Aldosterone/renin ratio B. Alkalne phosphatase C. BUN/creatnine ratio D. PT time E. Urine sodium excretion

C. BUN/creatinine ratio

43 year old man. 6 month history of intermittent abdominal pain that is associated with nausea. The patient describes episodes of dull epigastric pain that are usually worse 15-30 minutes after meals and last for a few hours. Pain is not relieved with antacids but improves when leaning forward. He has also had occasional diarrhea. The patient has lost 15 lb over the last 12 months. Five years ago, hospitalized for 3 days with acute abdominal pain. Patient smokes a pack of cigarettes a day and consumes alcohol almost daily. Which of the following will most likely indicate a diagnosis for this patient's condition? A. Anti-tissue transglutaminase antibodies B. Cancer-associated antigen 19-9 levels C. CT scan of the abdomen D. Mesenteric angiogram E. Serum lipase levels F. Upper GI endoscopy

C. CT scan of the abdomen - This is chronic pancreatitis

55 year old male. face and arm swelling that is worse when he wakes in the morning. 10 pound unintentioanl weight loss over the past 3 months, new onset dypsnea. Has hypertension, smoked for 40 years. His face is plethoric and dark-appearing. Veins on the anterior chest wall appear engorged. Soft, nontender nondistended abdomen. No edema in lower extremties. What is teh next best step? A. echo B. Ultrasound of upper extremities C. chest x-ray D. urine for 24 hour protein excretion E. Serum albumin level

C. Chest x-ray - These signs and suymptosm are consistent with superior vena cava syndrome - Signs/symptoms include swelling of the head, neck and arms; dypsnea, venous congestion - Malignancy is the most common cause of obstruction adn with his smoking history, likely to find a lung cancer

68 year old woman with T2DM hospitalized fo acute STEMI. Undergoes immediate cardiac cath and successful stenting. On the third day of hospitalization, she complains of vague abdominal pain. Lungs clear, heart sounds normal. There is bluish dicoloration of her right great toe and all the toes on her left foot. What is the most likely cause of this patient's symptoms? A. Arterial vasospasm B. Autoimmune vasculities C. Cholesterol embolism D. Reaction to iodine contrast E. Right to left shunt F. Side effect of lisionpril

C. Cholesterol embolism - Skin manifestations are the most common complications and include cyanotic toes with intact pulses ("blue toe syndrome"), livedo reticularis (painless purple mottling), gangrene and ulcers

35 year old man comes with SOB that started 2 days ago. Dry cough. Hx of hypertension. Ran out of meds 3-4 days ago. Smokes a pack a day and drinks 4-5 beers daily. Last drink 4 days ago. Basically, put in ICU and started on IV furosemide and nitroprusside. Next morning, nurse finds him confused and agitated and he has a generalized tonic clonic seizure. What is this? A. Alcohol withdarwal B. Aoritc dissection C. Cyanide poisoning D. Excessive hypotensive response E. Hypertensive encephalopathy F. Ischemic stroke G. severe hyperkalemia

C. Cyanide toxicity - This is not alcohol withdrawal because bis last drink was 4 days ago - Prolonged infusions, higher doses of nitroprusside can cause cyanide toxicity

65 year old man with fever, chills and a productive cough for the past 3 days. Shortness of breath and right-sided chest pain that is worse with deep breathing. Also has coronary artery disease, type 2 diabetes, and hyperlipidemia. Takes aspirin, atrovastatin, metformin and sitagliptin. Temp is 102.5, BP 95/60, pulse 112, respirations 24. O2 sat 96% on RA. Labs show leukocytes 16,500, BUN 48, creatinine 2.0. Chest x-ray shows right lower lobe infiltrate. Blood cultures obtained and IV fluids started. What next? A. Discontineu aspirin B. Discontinue atorvastatin C. Discontinue metformin D. Start IV insulin infusion E. Start lisinopril F. Start zoledronic acid

C. Discontinue metformin - He has renal injury - Metformin is contraindicated in renal injury, at least until it improves

A 45 year old white male presents to the ER after an episode of syncope. His medical history is significant for an upper respiratory infection one week ago. On physical examination, his temp is 99, pulse is 90, blood pressure is 100/60, and respirations are 13. HIs neck veins are distended, and his heart sounds are distant. His lungs are clear to auscultation bilaterally. Chest x-ray reveals small bilateral pleural effusions and an enlarged cardiac silhouette. which of the following EKG findings is fairly specific for his condition? A. Prolonged PR interval B. Presence of F waves C. Electrical alternans D. Presence of delta wave E. New-onset right bundle branch block

C. Electrical alternans - Beck's triad: distant heart sounds + hypotension + distended neck veins = pericardial tamponade/effusion - often secondary to viral pericarditis - You see electrical alternans with this

34 year old man comes in with frequent headaches. Diagnosed wtih migraines a year ago but has had no relief with medications. Has had high blood pressure on severeal checks, but not other PMH. The family history of hypertension or cardio disease. Blood pressure 185/105 pulse 88 and regular. A fourth heart sound is heard. A continous murmur is noted thrughout the thorax in multiple areas. What is most likely to be found on chest xy-ray? A. Curvature of the spine in 2 planes B. Diffuse vascular calcfiications C. Erosions of the inferio costal surfaces D. Prominent right atrial contour E. Upturning of the cardiac apex

C. Erosions of the inferior costal surfaces - This is coarctation of the aorta - Enlarged intercostal arteries erode the ribs A is turner syndrome, which can also cause coarctation, but is only seen in women - B is just premature atherosclerosis - D is Ebstein anomaly - E is Tetralogy of fallot ("the boot"), presents with cyanosis early in life

Which of these is a indicator of pending respiratory failure in an asthma exacerbation? A. Hypoxemia evident on ABG B. Lung hyperinflation noted on chest x-ray C. Normal PaCO2 on arterial blood gas D. Tachycardia E. Tachypnea

C. Normal PaCO2 on arterial blood gas - Normally, in an asthma exacerbation, the patient will be breathing faster, breathing off their CO2 and causing a respiratory alkalosis - As they continue to breath hard, they can't keep up with adequate ventilation, they get muscle respiratory fatigue and severe air trapping, causing a normal CO2

64 year old woman with headache and nausea for 2 days. End-stage renal disease due to advanced diabetic nephropahty and recently started hemodialysis. Has anemia from CKD and recently started erythropoietin and iron. Has mild hypertension, osteoporosis and hypothyroidism. Blood pressure 210/121, pulse 76,. What is the most likely cause of the current condition? A. Cerebellar hemorrhage B. Cerebral venous thrombosis C. Erythropoietin therapy D. Fibrinoid necrosis of renal arterioles E. Osmotic shifts during hemodialysis

C. Erythropoietin therapy - She has symptoms of acute hypertensive crisis - Some patients on erythropoietin can develop worsening hypertension - Also causes headache and nausea - Osmotic shifts can cause headache and nausea, but doesn't worsen hypertension - Fibrinoid necrosis is a consequence of uncontrolled hypertension, not a cause

43 year old man. progressive SOB and fatigue for the past several weeks. Two years ago, dx with alcoholic cirrhosis and esophageal varices. Several months ago, massive ascites. Current meds include fuosemide, spironolactone and nadolol. No alcohol for the past 2 years. No change in dypsnea on lying down or moving into an upright position. Neck veins are flat. Heart sounds normal, no murmurs or gallop. Dullness and decreased breath sounds are present on the right. Left sided breath sounds normal. Shiftness dullness, moderate distension of abdomen. 1+ bilateral lower extremity pitting edema. Most likely cause? A. Alcohol induced cardiac dysfunciton B. Decreased diaphgramatic excursion C. Fluid passage through diaphragmatic defects D. Intrapulmonary vascular dilations E. Medication adverse effects F. recurrent variceal bleeding

C. Fluid passage through diaphragmatic defects - This is hepatic hydrothorax - pleural effusion not due to cardiac or pulmonary problem - Generally results in transudative pleural effusion and is thought to occur due to small defects in the diaphragm - No other finidngs to suggest heart failure - D is hepatopulmonary syndrome - usually have increased dyspnea while upright or oxygen desaturation while upright

63 year old woman. leg swelling that is espeically bothersome in the evening. Symptoms have gradually worsened over teh last year. Has hypertension and OSA. Smoked daily for 30 years. Blood pressure 160/90. BMi 32. Bilateral 2+ pitting edema in lower extremitie to midshin with dilated and tortuous superficial veins. JVP 2 cm above. Small ulcer noted on left medial ankle. All peripheral pulses palapble. Which of the following is most likely to relieve this patient's current symptoms? A. daily furosemide B. Dietray sodium restriction C. Frequent leg elevation D. Improved control of hypertension E. Smokign cessation

C. Frequent leg elevation - this is NOT heart failure - this is chronic venous insufficiency - Typically worse in the evenings or following prolonged standing and improves after walking or leg elevation - abnormal venous dilation, skin ulceration.. - Heart failure would have dypsnea and elevated JVP

62 year old woman comes to office with urinary frequency and burning during urination. Long standing diabetes and hypertension. Creatinine is 1.1. Urinalysis shows negative glucose, negative ketones, positive nitrites, 2+ protein, 20-25 white blood cells, 3-5 red blood cells. Given a course of TMP-SMX. Two weeks later, UA reveals 2+ protein, but no nitrites, white blood cells, or red blood cells. Repeat serum creatinine is 1.1. Symptoms have resolved. What is responsible for her persistent urinalysis abnormality? A. Acute inflammatory infiltrate in the kidney interstitium B. Atherosclerotic narrowing of the renal arteries C. Glomerular basement membrane changes D. Insoluble crystal precipitation in the tubular lumen E. Retrograde passage of urine into the renal pelvis

C. Glomerular basement membrane changes - Natural history of diabetic nephropathy

48 year old woman. Day of fever and skin rash. Eight months ago, treated for ovarian cancer. Recent imaging study showed metastatic lesions to the liver, and a new chemo regimen was started a week ago. Temp is 102, blood pressure is 110/60, pulse is 114. The trunk has several, indurated, nontender macules and papules. Some of the lesions have become gangrenous ulcers. What is the most likely cause of these skin lesions? A. Cutaneous metastasis B. Drug-induced hypersensitivity reaction C. Gram negative bacteremia D. Latent virus reactivation E. Pyoderma gangrenosum

C. Gram negative bacteremia - She has Psudomonas from being immunocompromised and now has ecthyma gangrenosum - symptoms progress rapidly (she has gangrenous ulcers within a day)

A 35 year old man comes to the physician due to a 1 month history of diarrhea, abdominal distension, and flatulence. Over this period, he has lost 15 lb and developed night sweats and occasional arthralgias. There is no history of recent travel. On physical examination, his temp is 99.5, blood pressure 112/74m respirations 16, pulse 104. PE shows enlarged, nontender cervical and inguinal lymph nodes. A small ulcer is seen on the buccal mucosa. He has no skin rashes or joint effusions. What is the next best step? A. ANA screen B. Colonoscopy with biopsies C. HIV testing D. Thyroid function tests E. Tissue transglutaminase antibodies

C. HIV testing - Can have very nonspecific symptoms at first - Celiac could cause the GI symptoms but it wouldn't explain the lymphadenopathy and night sweats

20 year old man from northern europe comes to ED with 2 month history of intermittent right upper quadrant pain. no signficant PMH. mother had gallbladder removed at a young age. Abdominla ultrasound shows cholelithiasis. Lab results show hemoglobin of 10.2. Which of the following additional laboraotry findings does this pesron likely have? A. ABsence of CD55 in red blood cell membranes B. high hemoglobin A2 and hemoglobin F levels C. High MCHC D. High MCV E. Low serum iron and ferritin levels

C. High MCHC - This is herediatry spherocytosis

38 year old woman. fatty food intolerance. mild RUQ tenderness on exam. Ultrasound reveals multiple gallstones with no evidence of cholecystitis and an ill-defined intrahepatic cyst with a thick wall. CT abdomen reveals a well-defined cyst with eggshell calcification. No PMN. No drugs, no surgeries. has not traveled outside the US. She has had 2 pet dogs at home for 10 years. Parents emigrated from Peru many years ago. Most likely cause of the cyst? A. Amebic liver abscess B. Cysticercosis C. Hydatid cyst D. Pyogenic liver abscess E. Simple hepatic cyst

C. Hydatid cyst - This Echinococcus - from the dogs - "eggshell calcification" is highly suggestive of a hyatid cyt

35 year old woman. 5 month history of episodic retrosternal pain that radiates to the interscapular region. Episodes typically last 15 minutes, precipitated by emotional stress and hot or cold food. Regurgitates food intermittently. ECG normal. Chest XR, upper GI endoscopy and echo are normal. What is the next best step? A. 24 hour ph monitoring B. coronary angiogram C. CT scan of the chest D. Esophageal motility studies E. Pulmonary V/Q scan

D. Esophageal motility - This is diffuse esophageal spasm

a 45 year old woman comes to the physician complaining of fatigue, weakness, and diffuse bone pain. She was diagnosed with celiac sprue 5 years ago and admits to being noncompliant with her diet. She takes over the counter folic acid and iron. Lab results show normal calcium, low phosphate, high parathyroid, and high alk phos. Which of the following is responsible? A. Accelerated focal bone remodeling B. Defective formation of collagen C. Impaired osteoid matrix mineralization D. Low bone mass with normal mineralization E. Lytic lesions due to malignant cell infiltration

C. Impaired osteoid matrix mineralization - This is osteomalacia, can commonly happen with celiac - They become vitamin D deficient - leads to decreased calcium and phosphate absorption - Hypocalcemia and low vitamin D then trigger secondary hyperparathyroidism, causing high parathyroid and high alk phos - This secondary hyperparathyroidism then brings calcium levels to normal (or nearly normal) and also increased urinary phosphate excretion, worsening the hypophosphatemia, causing a marked hypophosphatemia compared to a normal or slightly low calcium

49 year old man. 3 month history of fatigue. Diffuse joint pain, finger swelling, difficulty gripping objects with his right hand. Has poorly controlled hypertension despite being compliant with medications. Facial features appear coarse, palms are sweaty, and have a doughy feel. The skin is oily. Multiple skin tags are noted, particularly on the neck area. What is the best next step in evaluation? A. Fasting GH level B. GH level following an oral glucose load C. Insulin-like growth factor 1 level D. MRI of the pituitary gland E. Thyrotropin-releasing hormone stimulation test

C. Insulin-like growth factor 1 level - This is acromegaly - Can't use GH levels, because they can fluctuate a lot during the day

58 year old male with a history of extensive alcohol use admitted witth diagnosis of decompensated liver cirrhosis and ascites. On admission, he is mildly metabolically acidemic. 3 days later, he has a metabolic alkalosis. Which of the following best explains the acid-base status change in this patient? A. Acute kidney injury B. Bowel ischemia C. Loop diuretic therapy D. opioid medication use E. Right lower lobe atelectasis

C. Loop diuretic therapy - Causes a metabolic alkalosis = Frequently administered to cirrhotic patients with asictes and volume overload

31 year old woman with a 6 week history of periorbital edema and abdomoinal distension. Her temperature is 98.9, blood pressuure 125/75 pulse 80 and respirations are 14. Exam shows moderate asictes and lower extremity edema. Urinalysis shows proteinuria. Renal ultrasound normal. A renal biopsy is performed. The patietn is started on diuretics and salt and protein intake is restricted. The edema begins to improve. Howevere, the patietn suddenly develops severe right-sided abdominal pain, fever, and gross hematuria. What is most likely to be diagnosed by renal biopsy? A. FSG B. IgA nephropathy C. Membranous glomerulopathy D. Minimal change disease E. Systemic amyloidosis

C. Membranous glomerulopathy - Her presentation is consistent with renal vein thrombosis - Is a complication of all types of nephrotic syndrome, but is most commonly seen with membranous glomerulopathy

18 year old male going to central african for a class trip. Diagnosed with Crohn disease 3 years ago but asymptomatic on methotrexate and adalimumab. He had meningococcal vaccine at age 11. No vaccines since then, but patient believes he receieved all the recommended childhood vaccines before that. What vaccine should be recommended for this patient? A. Intranasal influenza B. Measles, mumps, rubella C. Meningococcal booster D. Varicella E. Yellow fever

C. Meningococcal booster - Booster vaccine should be provided at age 16 ideally (up to age 21) if the primary vaccine was age age <16 - Also, booster recommended before travel to highly endemic environments, such as most of Africa and Saudi Arabia - Yellow fever would be recommended for most people before travel to central africa, howevere contraindications include: allergy to eggs, AIDS (CD4 <200), recent stem cell transplant, certain immunodeficiencies, and treatment with immunosuppressants (such as in this patient)

A 65 year old man with a history of COPD comes to the ED due to gradually worsening SOB and wheezing for the past several days. The patient also has a productive cough with mucoid sputum but no fever or chest pain. his other medical problems include hypertension and hypercholesterolemia. The patient has a 35 pack year smoking history and quit 5 years ago. Temp is 98.8, BP 130/80, pulse 114 and irregular. Pulse ox is 86% on RA. Physical exam reveals moderate respiratory distress, bilateral expiratory wheezing, and distant heart sounds. ECG shows an irregular narrow complex tachycardia with 3 different P wave morphologies and a variable PR interval. Serum electrolytes are within normal limits. Which of the folloiwng is the best next step in management of this patine'ts cardiac arrhythymia? A. Antiarrhythmic medication to convert to normal sinus rhythm B. Anticoagluation and a calcium channel blocker C. Monitoring while treating COPD exacerbation D. Rate control with a nonselective beta blocker E. TEE and cardioversion

C. Monitoring while treating COPD - This is Multifocal atrial tachycardia (MAT) - Commonly seen in elderly patients with acute exacerbation of underlying pulmonary disease - Best treatment is management of the underlying problem

A 28 year old woman female complaints of recurrent nasal discharge and increasing nasal congestion. She has a constant sensation of dripping in the back of her throat and states that food has tasted bland to her recently. She is known to have sickle cell trait. She came to the emergency department for severe wheezing after taking naproxen one year ago. She has no history of head trauma. She does not smoke cigarettes, but does smoke marijuana occasionally. Which of the following is the most likely diagnosis? A. Angiofibroma B. Inverted papilloma C. Nasal polyp D. Perforated nasal septum E. Pyogenic granuloma

C. Nasal polyp - The patient's history of wheezing following naproxen is suggestive of aspirin exacerbated respiratory disease (AERD), a condition commonly associated with the development of nasal polyps - AERD: asthma + chronic rhinosinuisitis with nasal polyps + bronchospasm or nasal congestion following ingestion of aspirin or NSAIDs

58 year old man. 1 year hisotry of diarrhea. Stools are watery and accompanied by abdominal cramps. No fever, BRBPR, or foul-smelling stools. Also experienced frequent episodes of dizziness, flushing, wheezing and a feeling of warmth. HE is depressed about his illness and feels hopeless. He appears ill. Cardiac auscultation shows a 2/6 systolic murmur over the left lower sternal border thar increases with inspiration. Abdominal examination shows hepatomegaly 3 cm below the right costal margin. Patient is at risk of developing a deficiency of which of the following vitamins or minerals? A. Calcium B. Iron C. Niacin D. Vitamin A E. Vitamin C

C. Niacin - CARCINOID SYNDROME - They get niacin deficiency

89 year old woman with episodic skin discoloration over the last several months. reports dark purple area on her hands and forearms without associated pain or itching. PE shows thin, hyperpigmented skin with several flat, dark purple ecchymotic areas over the dorsum of both forearms. LAbs show normal platelets, normal fibrinogen, normal PT, normal INR and normal PTT. What is the most likely cause? A. Bone marrow failure B. Lupus anticoagulant C. Perivascualr connective tissue atrophy D. Pool platelet adhesion E. vitamin K deficiency

C. Perivascular connective tissue atrophy - This is senile pupura - Common in old people, doesn't require any additional treatment

27 year old man comes to the physician for a follow up for hypertension. Feels healthy, no complaints. two weeks ago, found to high HBP during emergency visit for a sports injury. Strong family hisotry of hypertension and storke. Blood pressure 155/97 and pulse 78. PE and lab workup unremarakble. He is instructed to follow a low salt diet and started on low dose hydrochlorothiazide. Two weeks later, the patient comes to the ED complaining of muscle cramps, weakness and palpitations. His blood pressure is 150/93, pulse is 82. Lab shows a potassium of 2.5. WHat is the next best step? A. CT of abdomen B. Dexamethasone suppression C. Plasma renin activity and aldosterone concentration D. Serum thiazide level E. Urine potassium excretion by 24 hour collection

C. Plasma renin activity and aldsoterone concentration - Young patient with strong family history - suspicious for secondary cause of hypertension - Easily induced hypokalemia after starting a thiazide diuretic - think primary hyperaldosteronism - Thiazide diuretics themselves can cause hypokalmia - howevere, low dose thiazides would not be expected to cause this level of hypokalemia

A 35 year old African American woman comes to the clinic due to exertional shortness of breath. Exercise tolerance a year ago was 7 blocks, but now can't even walk a block. Feels lightheaded. Has been treated for Raynaud's phenomenon and fingertip ulcerations for the last 2 yeras. Also has severe heartburn controlled with high-dose pantoprazole. No cigarettes or alcohol. Pulse ox 95%. BMI 23. What is most expected on physical examination? A. Delayed femoral pulses bilaterally B. Ejection-type systolic murmur radiating to the carotids C. Right ventricular heave D. third heart sound at the apex on end-expiration E. Wide and fixed splitting of the second heart sound

C. Right ventricular heave - Most likely has CREST syndrome: calcinosis + Raynaud + esophageal dysmotility + scleroderma + telangectasias - PAH is very common with this - Lung biopsy: arterial intimal hyperplasia

34 year old man with no significant PMH comes in endocarditis. Started on empiric IV vanc. Blood cultures end up growing STrep mutans taht is very sensitive to penicillin. What do you do next? A. Continue current regimen B. Switch to IV aminoglycoside C. Switch to IV ceftriaxone D. Switch to oral amox/clav E. Switch to oral penicillin V

C. Switch to IV ceftriaxone - in these strains that are susceptible you can treat with penicillin or ceftriaxone - BUT it has to be IV - it can't be oral, endocarditis is too dangerous

28 year old woman. no symptoms. 2 previous miscarriages. mom has hypothyroidism. she has symetrically enlarged, nontender, firm thyroid gland. TSH is elevated, free T4 is normal. What antibody is associated with her condition? A. Anticardiolipind antibody B. Antimitochondrial antibody C. Antithyroid peroxidase antibody D. Thyroid stimulating immunoglobulin E. TSH-receptor blocking antibody

C. antithyroid peroxidase antibody - She has subclinical hypothyroidism - Anti-Tpo antibodies are the most common - high titers of these antibodies are also associated with increased risk of miscarriage

41 year old man, 4 days of rapidly worsening dyspnea, nonproductive cough, fever and chills. Developed acute liver failure due to Wilson's disease and had a liver transplant 5 months ago. Takes several immunosupressants, but no antibiotics. temp is 102, pulse 112, O2 sat 82% on RA. Pulmopnary examination receals bilateral crackles. Leukocyte count is 12,000 and serum LDH is 330. Chest x-ray reveals bilateral, diffuse interstitial infiltrates. An adequate sputum sample cannot be obtained. which of the following tests is most likely to yield the diagnosis in this patient? A. Beta D glucan assay b. blood culture C. bronchoalveolar lavage D. high resolution CT scan of the chest E. nasopharyngeal swab culture F. urine legionella antigen

C. bronchoalveolar lavage - PJP pneumonia

54 year old man woke up in middle of night with substernal discomfort, describes as burning sensation. Also has left-sided neck pain and feels sweaty and SOB. never had similar pain before. PMH significant for T2DM and hypertension. Also has a 30 pack year smoking history. What physical exam findings is most commonly associated with this patients' presentation? A. Ejection-type systolic murmur B. Fixed splitting of S2 C. Fourth heart sound D. Friction rub E. Pulsus paradoxus

C. fourth heart sound - This is an acute MI - can often hear an S4 during the acute phase of an MI

35 year old male with HIV who has vision loss. Funduscopy shows pale, widespread, peripheral retinal lesions and central necrosis of the retina. What is the most likely organism? A. Pseudomonas B. CMV C. HSV D. Candida albicans E. Epstein Barr virus

C. herpes simplex - CMV is a very common cause of visual loss in HIV patients, but funduscopy shows fluffy or granular retinal lesions located near the retinal vessels and associated hemorrhages

40 year old man. comes in after not seeing a physician for many years. No medical conditions. Lately has been feeling fatigued and occasional muscle aches. Gained 9 lb over teh past year. No smoking. Vitals normal, BMI 26. Serum cholesterol high, LDL high, triglycerides high. What next? A. begin atrovastatin therapy B. inquire about anabolic steroid use C. obtain thyroid function studies D. prescribe ezetimibe therapy E. start gemfibrozil therapy F. test for LDL receptor mutation

C. obtain thyroid function studies - he has symptoms of hypothyroidism which can cause lipid abnormalities

72 year old woman. poorly controlled T2DM. comes to clinic 1 week after being discharged from hospital. Admitted there for pyelonephritis secondary to a multi-drug resistant organisms and given several days of IV antibiotics. Serum creatinine on admission was 2.1. Now it is 4.9. Urinalysis reveals rare epithelial casts and no white blood cells. FENa is greater than 2%. What antibiotic did she most likely receive? A. Nafcillin B. Vancomycin C. Levofloxacin D. Amikacin E. Doxycycline F. Azithromycin

D. Amikacin - Amikacin and vancomycin are 2 common causes of acute renal failure on this list - However, she probably was infected with a gram-negative rod which Amikacin would be used for - Vancomycyin would usuallly be used for MRSA, which is an uncommon cause of pyelonephritis

27 year old woman compalining of joint pain. Began 10 day sago and consists of bilateral pain in the MCP, PCP, wrists, knees and ankles. Describes joint stiffness upon awakening in the morning for 10-15 minutes. Also has had associated fatigue and a few episodes of loose bowel movements associated with mild skin itching and patchy redness. No medication conditions. The patient is married and has 2 children. Works as a elementary school teacher. On exam, tenderness of involved joints without swelling or redness. What is most likely in this patient? A. Anti-cyclic citrullinated peptide antiboides B. Anti-double stranded DNA antibodies C. Antinuclear antibodies D. Anti-parvovirus B19 IgM antiboides E. Anti-streptolysin titer F. Cryoglobulin levels G. Rhumeatoid factor

D. Anti-parvovirus B19 IgM antibodies - Parvovirus - School teacher - symmetric arthritis of the hands, knees and ankle joints

58 year old woman. Had a renal transplant 2 years ago. Had been feeling well until the past several weeks when she noticed decreased urine output and mild fatigue. Current meds include diltiazem, apixaban, prednisone, tacrolimus and mycophenolate sodium. Blood pressure 152/86, pulse 82, respirations 12. No abdominal tenderness. Lab results show a creatinine of 2.3, which is increased from her baseline of 1.4 that was last checked 2 months ago. Urinalysis shows trace protein but not much else. Blood tacrolimus level is moderately elevated. Transplant ultrasound reveals no hydronephrosis. What is responsible for the patient's worsneing renal function? A. Acute interstitial nephritis B. Acute tubular necrosis C. Antibody-mediated rejection D. Anteriolar vasconstriction E. Polyoma virus reactivation

D. Arteriolar vasoconstriction - Acute calcineurin inhibitor renal toxicity - cyclosproine and tacrolimus can cause this - They have adverse effects such as vasoconstrictive properties

34 year old man. occasioall dizziness and palpitations in crowded spaces. Report high level of stress at work. no medical history. Occasional alcohol, no drugs or tobacco. Blood pressure and pulse are normal. Chest exam normal. Liver 8 cm, spleen not palpable. No cervical lymphdenopaty or skin rash. Hemoglobin nornal, platelets low, leukocytes noral. Peripheral smear confirms a reduced platelet count wihtout clumping or malignant cells. What is teh most approrpiate next step? A. Blood folate level B. CT scan of the abdomen C. EBV titers D. HIV antibody testing E. RPR test F. Schilling test

D. HIV antibody test - Isolate thrombocytopenia without anemia or leukopenia suggests ITP - Any patient with ITP should be tested for both HIV and hepatitis C

56 year old man. chest pain for an hour accompanied by sweating and left arm numbness. Symptoms improved wtih aspirin and sublingual nitroglycerin. History of GERD, T2DM and smoking. ECG shows normal sinus rhythm with T wave inversion in leads V1-V4. Troponin normal. What next? A. Admknister lidocaine B.Exercise stress testing C. IV alteplase D. IV heparin E. Oral proton pump inhibitor

D. IV heparin infusion - This is unstable angina - nitrates, beta blockers, antiplatelet therapy, anticoagulation, statin therapy, coronary reperfusion

A 34 year old man is brought to the ED due to several hours of confusion. His wife reports that he has fever, malaise and cough for the past 2 days. A year ago the patient required prolonged hospitalization and extensive surgery for multiple gunshot wounds to the abdomen. He has no other medical problems. The patient does not use tobacco, alcohol or drugs. He has no history of recent travel. Temp is 104.9, blood pressure is 80/50 and pulse is 100. Dullness to percussion and crackles over the left lower chest are present. Cardio exam revecals normal first and second heart sounds and bounding peripheral pulses. The abdomen has several well-healed surgical scars. IV fluids and broad-spectrum antibiotics are started. The next day, blood cultures grow gram-positive cocci. Which of the following is the most likely underlying mechanism leading to this patient's clinical presentation? A. Complement deficiency B. DEstruction of CD4+ cells C. Immunoglobulin A deficiency D. Impaired antibody-facilitated phagocytosis E. Impaired B cell isotype switching F. Impaired chemotaxis G. Impaired oxidative burst

D. Impaired antibody-facilitated phagocytosis - This man likely had a splenectomy for the gunshot wounds - He is at risk for encapsulated infections such as H. flu and strep pneumo due to deficits in antibody response and antibody-mediated phagocytosis

82 year old man evaluated for heart murmur. says he feels healthy and walks severael miles per day. Echo shows calcified aortic valve but no stenosis, but severe aortic regurgitation. The left ventricle is dilated wtih an ejection fraction of 63%. No other valve abnormalities. What is the compensatory mechanism that explaisn why this patient is asymptomatic? A. Decreased forward stroke volume B. Decreased left ventricular contractility C. Decreased left ventricular preload D. Increased left ventricualr compliance E. Increased total peripheral resistance

D. Increased left ventricular compliance - Regurgitation - bunch of blood is flowing backwrads - Causes the left ventricle to be dilated - The left ventricle adapts via hypertrophy - allows for increased compliance to accomade the additional volume and maintain cardiac output

16 year old boy high fever and sore throat. He has severe diffuse leg pain. Symptoms started abruptly 8 hours ago. Temp 102.4. Says he has "never felt as terrible.:" Diffuse erythema of the posterior pharynx without exudate. tonsils 2+ bilatearlly. Supple neck. No hepatosplenomegaly. B/l lower extremities are diffusely tender to palpation with mottling of the skin. Rapid strep and ifluenza are negative. What is teh next best step? A. CT scan of the neck B. Heterophile antibody test C. HIV antigen/antibody test D. Lumbar puncture E. Throat culture for diphtheria

D. Lumbar puncture - Meningococcal meningitis - Early meningitis can cause nonspecific symptoms such as fever, headache and vomiting - Non-suppurative meningitis is a rare presentation of meningococcal meningitis that may be mistaken for strep or viral pharyngitis - Often patients with phayngitis from meningococcus have rapid disease progression (<12 hours) and often dscribe their illness and "worst i have ever felt"

56 year old man with progressive urinary frequenty, urgency and hesistancy for the past several months. Also has new onset low back pain and perianal pain during ejculation. No dysuria or hematuria. Has smoked a pack daily for 30 years. Temp is 98.6, blood pressure 130/76, pulse 80. No suprapubic or CVA tenderness. Rectal examination shows increased anal sphincter tone and a smooth, slightly enlarged prostate. Urinalysis shows many leukocytes and no erythrocytes. Urien culture grows no organisms. serum PSA normal. Most likely cause? A. Bacterial sequestration in the epididymis B. Hyperplasia of prostatic stromal and epithelial cells C. Malignancy of prostatic glandular elements D. Noninfectious chronci prostate inflammation E. Resitant bacterial infection of the prostate F. Urothelial neoplasm arising from the bladder

D. Noninfectious chronic prostate inflammation - Chronic prostatitis - Diagnosis of exclusion

51 year old man. nausea and fatigue for the past several weeks. occaisonal episdoes of bilateral flank pain over the past severeal years and nocturia 2 or 3 times per night for the past 10 years. No change in appetitie or weight. Vitals normal except for BP 164/100. Liver is enlarged, and a mass if felt at the right flank on deep palpation. Prostate is mildly enlarged and nontender. No extremity edema. Low hemoglobin, really high BUN and creatinine. What is most likely? A. Bladder outlet obstruction B. Horseshoe kidney C. Nephorlithiasis D. Polycystic kidney disease E. Renal cell carcinoma

D. Polycystic kidney disease - Flank pain, hematuria, hypertension, palpable abdominal mass

64 year old woman bitemporal headaches for a week. Dizziness, blurry vision and tingling and numbness of her feet. No fever or jaw pain. Vital signs snormal. Moist mucous membranes, normal tympanic membranes, and no sinus or temporal area tenderness. Visual acuity decreased in both eyes, funduscopic examination reveals dilated, segmented and tortuous retinal veins. Facial sensation and strength are normal. B/l UE/LE muscle tsregnth normal. Decreased pinprick sensation and loss of ankle reflex b/l. What is the next best diagnostic study? A. Antineutrophil cytoplasmic antibodies B. HbA1c C. MRI of the brain D. Serum protein electrophoresis E. Temporal artery biopsy

D. Serum protein electrophoresis - This is Waldenstrom macroglobulinemia - hyperviscosity syndrome, neuropathy, bleeding, hepatosplenomegaly, lymphadenopathy - Monoclonal antibody: IgM - Rouleaux on peripheral smear

42 year old woman with 4 month history of hearburn. Describes a "sticking sensation" in her chest during meals. Dyspnea on exertion and joint pain in her hands and feet. Lung exam reveals bilateral end-expiraotry crackles. Endoscopic evaluation shows mild hyperemia in the distal esophagus. Esophageal manometry shows lack of peristaltic waves in the lower two thirds of teh esophagus and a significant decrease in lower esophageal sphincter tone. What is teh msot likely mechanism responsible? A. dysfunction of inhibitory neurons B. eosinophilic infiltration of esophageal mucosa C. loss of intramural neurons D. smooth muscle atrophy and fibrosis E. striated muscle inflammation

D. Smooth muscle atrophy and fibrosis - this is systemic scleroderma

68 year old woman with history of advanced COPD. Comes in with increased SOB and cough for 12 hours. 45 pack year smoking history. Pulse ox shows 94% on 2L oxygen. Given supplemental oxygen, IV antibitoics, methylprednisolone, and 2 treatments of atrovent. Following these measures, she is still dyspneic and using accessory muscles of respiration. ABG on 4 L shows ph 7.32, PCO2 60, PO2 52. What now? A. Decrease supplemental oxygen flow rate B. Increased supplemental oxygen flow rate C. Intubate and mechanically ventilate D. Start noninvasive positive-pressure ventilation

D. Start non-invasive positive-pressure ventilation - You can't decrease her oxygen 0 will just become more hypoxemic - You can't Increase her oxygen - will just make her not want to breathe off her CO2 and she will become more hypoxemic - Putting her on intubation is risky, because you might not be able to ever wean her back off of it given her disease state

32 year old man with 2 day history of fever, headache, malaise and myaglias. Family members say that he seems slightly confused. Recalls getting a tick bite 2 weeks ago while walking through the woods in Arkansas. Temp is 102. No rash, no lymphadenopathy, no cardio, no focal neurological deficits. Low leukocytes, AST 98, ALT 105. What is the most apporpriate next step in the patient? A. bone marrow biopsy B. Ceftriaxone C. Chloramphenicol D. doxycycline E. Erythromycin F. Hepatitis serology G. Lyme serology

D. doxycycline - This is NOT lyme disease - This is Ehrilichiosis- caused by lone start tick - Seen in southeastern and south central united states - Flu like illness, neurologic symptoms (confusion), rash is UNCOMMON - Can see elevated liver enzymes, leukopenia and thrombocytopenia - Empiric doxycycline while awaiting confirmatory testing - Ceftriaxone would be for Lyme disease - not usually found in Arkansas, doesn't usually cause high fever, or liver/count abnormalities

54 year old woman. fatigue, tingling in feet. no medical problems. no alcohol, drugs or tobacco. She has pale conjunctivae and a shiny tongue. Ankle reflexes decreased bilaterally. hemoglobin 7.6, MCV 120. She is greatest risk for which of the following long-term complications? A. acute myeloid leukemia B. aortic aneurysm C. cirrhosis D. gastric cancer E. glomerulonephritis F. t-cell lymphoma

D. gastric cancer - She clearly has B12 deficiency - Most common cause is pernicious anmiea - Over time, this causes intestinal-type metaplasia of the gastric mucosa increasing the risk of gastric cancer

58 year old man brought in after apparent suicide attempt. Found lethargic by his wife with a suicide note. Temp 100, blood pressure 76/40, pulse 40, respirations are 12. Exam shows diffuse bilateral wheezing. Extremities cold and clammy. ECG shows profound sinus bradycardia with first-degree AV block. Given IV fluids and atropine, but bradycardia and hypotension do not improve. In addition to ensuring adeuqate oxygenation, what is the most appropriate next step? A. Aminophylline B. digoxin-specific antibody C. dobutamine D. glucagon E. IV pacing

D. glucagon - These symptoms are suggestive of beta blocker overdose

38 year old woman. progressive muscle weakness over the last 6 months. can't climb stairs or comb hair. no associated muscle pain. diagnosed with hypothyroidism a year ago, taking levothyroxine. four months ago, developed acute back pain. x ray revealed demineralization of the vertebral bones. BMI 32. PE shows facial hirsutism and mild proximal muscle weakness in the extremities. No tenderness to palpation of muscles, no fasciculations. DTR and senosry normal. TSH normal. What is responsible for the patient's muscle weakness? A. electrolyte abnormalities B. mitochondrial dysfunction C. motor neuron disease D. muscle atrophy E. muscle inflammation F. peripheral nerve demylination

D. muscle atrophy - She has symptoms consistent with Cushing syndrome - Usually caused by exogenous steroids, but can also be caused by ACTH producing pituitary tumor, ectopic ACTH production or primary adrenal disease - Myopathy from cushing syndrome caused by muscle atrophy

How do ACEi and ARBs slow the progression of renal disease in a diabetic? A. Increased aldosterone activity B. Increasd renal arterial blood flow C. Increased renal tubular glucose absorption D. Reduced glomeular hydrostatic pressure E. Reduced renal tubular protein secretion

D. reduced glomerular hydrostatic pressure - Slow the progression of diabetic nephropathy primarily by blocking angiogtensin II-mediated renal efferent arteriole vasoconstriction

42 year old male. nonpruritic nonpainful skin lesion on the left upper arm that began 2 months ago. Emigrated from Southeast Asia. Vital signs normal. Skin exam shows a 4 cm well-circumscribed hypopigmented patch on the left upper arm with no sensation to pinprick. Ulnar nerve is thickened and tender at the left elbow. Tocuh and pain absent in the lef tulnar nerve distribution. How to confirm the diagnosis in this patient? A. anti-borrelia antibody asssay B. koh preparation of skin scrapings C. nerve conduction studies d. skin biopsy from the edge of the lesion E. treponemal serologic testing F. tuberculin skin testing

D. skin biopsy from the edge of the lesion - This is Leprosy - Macular anesthetic skin lesions with raised borders - Nodular, painful nearby nerves with loss of sensory/motor function - Full-thickness biopsy of skin lesion

Patient has orthostatic hypotension, syncopal episode. 2 days of diarrhea. What is the most sensitive indicator of this patient's current condition? A. Decreased hematocrit B. Decreased plasma renin activity C. Decreased serum sodium D. Decreased urine osmolality E. Decreased urine sodium F. Increased BNP G. Increased serum potassium H. Increased urine urea nitrogen

E. Decreased urine sodium - He is dehydrated so aldosterone will be stimulated to maintain blood pressure - Aldosterone holds onto salt and gets rid of potassium so salt will be low in the urine

24 year old indian man. returned recently from india where he was visiting family. 3 days of left upper quadrant abdominal pain. abdominal ultrasound shows a spleen that is slightly enlarged with a hypoechoic wedge consistent with an infarction. what test should you do next? A. 24 hour ECG monitoring B. Bacterial blood cultures C. Bone marrow biopsy D. Factor V leiden mutation testing E. Hemoglobin electrophoresis

E. Hemoglobin electrophoresis - This is a splenic infarction from sickle cell disease - Certain stressors, such as flying at high altitudes, can precipitate this

A 22 year old woman comes to the ED due to continuous gum bleeding after a mounth injury a few hours ago. The patient has a history of bruising with minor trauma and of heavy menstruation with crampy pain. She has no other medical conditions. The patient does not use tobacco, alcohol or illicit drugs. Her mother also had a history of "bleeding issues." Blood pressure is 120/70, pulse 80. Oropharyngeal examination shows blood oozing from a gum abrasion. A fading ecchymosis is present on the right calf but there are no other skin abnormalities. The remainder of the physical examination is normal. Lab results show a normal PT but prolonged PTT. What is the most likely cause? A. Abnormal activation of coagulation and fibrinolysis B. Decreased factor VIII production C. Factor VIII inhibitor production D. Immune mediated platelet destruction E. Impaired platelet endothelial binding F. Reduced gamma carboxylation of vitamin K dependent clotting factors

E. Impaired platelet endothelial binding - This is Von Willebrand disease - Many patients are asymptomatic but a minority have easy bruising, skin bleeding, and mucosal bleeding (Ex. menstruation) A) is DIC - both prolonged PT and PTT B) Hemophilia A - causes much more severe bleeding problems C) form due to autoimmune diseases, malignancy, or pregnancy; much less common than VWF and are not generally associated with a family history of bleeding E) Immune thrombocytopenia - lab exam shows thrombocytopenia with normal PT and normal PTT F) VItamin K deficiency/antagonists (warfarin) would prolong INR and PT, with normal PTT

56 year old man. severe crushing midsternal chest pain accompanied by diaphoresis and shortness of breath. Had been having exertional angina for several months and 10 days ago, underwent coronary angiography that revealed a 90% stenosis of the proximal left anterior descending artery. Drug-eluting stent was placent. Echo was normal at discharge. The patient is currently unemployed and uninsured. ECG reveals sinus tach with 2mm ST elevations in leads V1-V4. What is the most likely cause of the current condition? A. Acute pericarditis B. Aortic dissection C. Atherosclerotic plaque rupture D. Left ventricular aneurysm formation E. Medication nonadherence

E. Medication nonadherence - Stents are thrombogenic, and the risk after getting one is mitigated by the administration of dual antiplatelet therapy with aspirin and clopidogrel - Poor adherence to this, a concern for people with limited access to medical care, is the most common cause of stent thrombosis, whcih is likely what is going on

63 year old man. severeal weeks of vague abdominal pain and increased fatigability. No PMH. smokes a pack of ciagrettes a day and consumes 5-6 beers on the weekend. PE shows mildly distended soft abdomen. The liver edge is hard and palpated 5 cm below the costal margin. Trace ankle edema is present. Labs show low hematocrit, low MCV, mildly elevated AST and ALT and elevated alk phos. Chest x-ray shows a small left-sided pleural effusion. Fecal occult blood test is positive. What is the cause of hepatomeagly in this patient? A. Alcoholic cirrhosis B. Autoimmune hepatitis C. Hemochromatosis D. Left ventricular failure E. Metastatic disease F. Nonalcoholic steatoheaptitis G. Portal hypertension H. Pulmonary hypertension

E. Metastatic disease - blood in stool + age + anemia + firm liver - most likely metastatic colon cancer to the liver

65 year old man with diabetes. Nausea, abdominal pain and early satiety for the past several months. No heartburn or epigastric pain but does have occasional vomiting. Diabetes complicated by retinopathy. Has had severeal episodes over the past few months where he has been hypoglycemic after meals despite decreases in premeal insulin dosage. What is the most helpful for treating this patient's condition? A. Diphenhydramine B. H. pylori eradication C. Lansoprazole D. Megestrol acetate E.Metoclopramide F. Ondansetron G. Promethazine H. Ranitidine

E. Metoclopramide - This is diabetic gastroparesis - Diffiulty with insulin absorption can cause frequent hypoglycemia - Metoclopramide usually used - small risk of extrepyramidal effects - You can also use erythromcyin - but usually only used for short time (<4 weeks) because the effects diminish over time

19 year old man. week of persistent dry cough that distrubs his sleep. Sore throat, headaches and fatigue. Yesterday noticed a rash on his arms. Temp 100. Mild pharyngeal erythema. No cervical lymphadenopathy. Cardiopulmonary exam is normal. Faint macular rash is present on extremities. Chest xray reveals increased interstitial markings and a small right sided pleural effusion. What is the most likely oragnism? A. EBV B. Influenza virus C. Legionella pneumophilia D. Moraxella E. Mycoplasma pneumoniae F. Parvovirus G. Strep pneumo

E. Mycoplams pneumoia - remember, atypical causes of pneumonia - Indolent headahce, malaisea, fever, peristent dry cough - Phayngitis, macular/vesicular rash

A 60 year old man comes to the urgent care clinic after being awakened by severe pain in his right great toe, which is suddnely swollen and very tender to the touch. He has also had occasional headahces and unbearable pruritus after a hot bath. the patient does not use alcohol, tobacco or illicit drugs. The liver span is 10 cm and spleen is palpable 2 cm below the costal margin. Aspiration of the MTP joint shows negatively birefringent crystals and many leukocytes, but no organisms. What is most likely? A. Chronic kidney disease B. Hemochromatosis C. Hyperparathyroidism D. Inherited enzyme disorder E. Myeloproliferative disorder F. Pernicious anemia G. Portal hypertension

E. Myeloproliferative disorder - Secondary cause of gout - Several featues of PV: puritus caused by hot baths, headaches and hepatosplenomegaly

32 year old woman. nagging dry cough over the last 8 weeks. Present during the day, awakens her at night. No shortness of breath, chest pain or wheezing. PMH significant for chronic rhinorrhea and occasional itching skin rash. No medications. Chest xr normal. One week of treatment with chlorpheniramine significnatly improves her symptoms. Decrease in which of the following is most likely responsible for her symptom relief? A. Acid aspiration B. Airway hyperreactivity C. Bradykinin production D. Bronchial inflammation E. Nasal secretions

E. Nasal secretions - Most likely has post-nasal drip from allergies

29 year old female wtih knee pain. 3 months. anterior aspect of left knee. Not improved with ibuprofen anymore. Worse when she walks up and down stairs. No medical problems. Smokes half a pack of cigarettes a day. BMI 26. Mild quadriceps atrophy with no visible bony deformity. No pain with palpation over teh patellar tendon and no evidence of swelling. What is most likely to be seen? A. Anterior translation of the tibia on the femur B. Apprehension with lateral translation of the patella C. Erythema and effusion of the medial joint line D. Inability to extend the knee E. Pain with isometric contraction of the quadriceps F. Tenderness at the tibial tubercle

E. Pain with isometric contraction -This is patellofemoral pain syndrome -Provocation of the pain with isometric contraction of the quadriceps strongly suggets this diagnosis

26 year old man undergoing chemotherapy. Comes in with fever and chills for the last 24 hours. No chest pain, SOB, abdominal pain, diarrhea or skin rash. temp is 102, other vital signs normal. Examination shows mucosal pallor. Mild gingival erythema but no evidence of bleeding. Labs show a leukocyte count of 690, hemoglobin 8.6, hematocrit 25%, platelets 74,000. Chest XR normal, UA normal. What is the best therapy for this patient? A. Acyclovir B. Ceftriaxone C. Ciprofloxacin D. Linezolid E. Piperacillin-tazobactam F. Vancomycin G. Voriconazole

E. Piperacillin-tazobactam - Febrile neutropena - absolute neutrophil count <1500 - No obvious source of infection, but commonly pseudomonas

34 year old woman. uncomfortable sensation of rapid heartbeat for the last few weks. NO associated chest pain or SOB. reports her anxiety level has been high recently. Has lost 5 lb during this time. PMH normal, no medications. Thyroid gland is diffusely enlarged and nontender. Lab studies show undetectable TSH and high free T4. Radioactive iodine is < 5% and anti-thyroid peroxidase antibodies are present in high titers. Next best step? A. Alprazolam B. Levothyroxine C. methimazole D. Prednisone E. Propranolol F. Radioactive iodine

E. Propranolol - She has symptoms and thyroid level consistent with hyperthroidism - Those antiboides are associated with chronic hashimotos' thyroiditis which is usually hypothyroidism - howevere, there is a variant called painless thyroidits - which is what she has - Usually they have a self-limited hyperthyroid phase, followed by a hypothyroid phase, which may persist or eventually return to a euthyroid stage - It does not require specific therapy, but can use beta blockers for symptomatic treatment

42 year old man comes in with periodic difficulty bleeding and wheezing. Visited an ENT for persistent nasal blockage 2 weeks ago. Patient was diagnosed with stable angina 6 months ago. Current medications include aspirin, diltiazem, atorvastatin, and albuterol as needed. What is teh most likely cause of his respiratory symptoms? A. Cell-mediated hypersensitivity B. Cytotoxic antibodies C. igE-mediated reaction D. Immune complex disease E. Pseudoallergic reaction

E. Pseudoallergic reaction - aspirin-exacerbated respiratory disease

37 year old woman. SOB and chest pain that started 2 days ago. Symptoms began suddenly and have worsened despite the use of acetaminophen. No fever. Diagnosed with MS 5 years ago. Is in a wheelchair due to spastic paraparesis. Reduced breath sounds and dullness to percussion at the left lung base. Chest x-ray shows a small left sided pleural effusion. NO infiltrates seen. What is the most likely cause of her pleural effusion? A. Aspiration pneumonia B. Congestive heart failure C. Hypoalbuminemia D. Malignant effusion E. Pulmonary embolism

E. Pulmonary embolism - Immobility - young, sudden onset of chest pain

A 53 year old man is admitted to the hospital with a 4 week history of fatigue and decreased exercise tolerance. Climbing 2 flights of stairs cuases significant dyspnea. He has had occasional palpitations for months but no chest pain. Past medical history is unremarkable, and a routine check-up 6 months ago was normal. He does not routinely take any medications. His blood pressure is 150/90 and pulse is 130 and irregular. Lungs are clear. ECG does not show clear P waves. Echo shows ejection fraction of 35%, moderate central mitral regurgitation, and left atrial and left ventricular dilation with global hypokinesis. which of the following interventions is most likely to restore left ventricular function in this patient? A. Coronary revascularization B. Decreasing afterload C. Inotropic medications D. Preload optimization E. Rate or rhythm control F. Valve surgery

E. Rate or rhythm control - This is a tachycardia-mediated cardiomyopathy - AF, atrial flutter, ventricular tachycardia, can all cause LV dilation and myocardial dysfunction - Most patients have signs/symptoms of heart failure - Aggressive rate control and rhythm control can potentially reverse this condition

66 yera old man brought to the ED with a lot of other people who developed rapid onset of SOB at a crowded shopping mall. Patinet reports smelling a faint, fruity odor prior to developing symptoms. The patient is diaphoretic and drooling. Pupils are pinpoint and unreactive. Diffuse rhonbhi and wheezing are present bilaterally. Which of the following would most likely confirm the diagnosis? A. Arterial blood gas test with carboxyhemoglobin B. BMP and plasma osmolality C. Geiger-Muller counter D. Methemoglobin level test E. Red blood cell acetylcholinesterase activity test

E. Red blood cell acetylcholinesterase activity test - This is organophosphate poisoning - Classically associated with farmers/field wokrers - But can also be used a terrorism nerve agent - sarin or soman gas

52 year old man with 24 hour history of pain and swelling in his right knee. No fevers, chills, or recent trauma to the knee. In addition, the patient has had constipation, excessive urination and fatigue for the past several months. PMH significant for kidney stone a year ago. No meds, tobacco, alcohol or drugs. Vital signs normal. Exam shows erythema, tenderness and swelling of the right knee. Labs show leukocytes of 13,000 and calcium of 11. What is likely to be found on synovial fluid analysis? A. Clear, transparent fluid B. hemorrhagic effusion C. Needle-shaped crystals D. Polymorphs filled with gram positive cocci E. Rhomboid shaped crystals

E. Rhomboid shaped crystals - He has symptoms of hyperparathyroidism - Pseudogout is a common complication of hyperparathyroidism with chronic hypercalcemia

52 year old man, history of alcoholic liver disease. admitted to hospital for fatigue and abdominal discomfort. no nausea, vomiting, constipation or diarrhea. Patient has chronic ascites treated with spironolactone and furosemide. Temp is 100.2, blood pressure normal, pulse 106. Alert, awake and oriented to time/place/person but can't perform a timed connect-the-numbers test. Moderate ascites with discomfort to palpation. Decreased bowel sounds. Abdominal x-ray shows dilated loops of large bowel with air in the colon and rectum. Most likely? A. Acute pancreatitis B. Alcoholic hepatitis C. Peptic ulcer perforation D. Small bowel obstruction E. Spontaneous bacterial peritonitis

E. SBP - Usally patients with cirrhosis are hypothermic, so this guy's 100 F temp is concerning - diffuse abdominal pain/discomfort, mental status changes, dilated loops of large bowel - Small bowel obstruction would give you dilated loops of SMALL bowel

48 year old woman, evaluation of elevated blood pressure. First noticed at dentist 4 weeks ago. Repeated home BP measurements around 150/90. No symptoms, no other medical conditions. Takes no medications. Dad has hypertension. Blood pressure is 152/88 twice in the office. BMI 28. normal physical exam. Normal ECG. Normal serum electrolytes and creatinine and no protein in the urine. Which of the following evaluations is most approrpiate in this patient? A. Assessment of left ventricular function B. Home sleep apnea testing C. Imaging for renovascular disease D. Measurement of plasma renin activity E. screening for diabetes

E. Screening for diabetes - Even in the absence of hypertension, screening is recommended for those age 45-70 with a BMI greater than or equal to 25

58 year old man with cirrhosis. Hospitalized for hepatic encephalopathy and SBP. Mental status improves, but has kidney injury that doesn't respond to albumin or IV fluids. What is hte most likely cause? A. drug-mediated renal tubular injury B. Glomerular immune complex deposition C. Intravascualr volume depletion D. Renal interstitial inflammation E. splanchnic arterial dilation

E. Splanchnic arterial dilation - This is hepatorenal syndrome

45 year old woman noted to have abnormal liver chemistries. no symptoms. Hypercholesteremia and takes statin for it. BMI 25. LAb results show high alk phos, normal AST, normal ALT. positive anti-mitochondrail antibodies. What next? A. Discontinue statin and repeat testing B. Measure serum alpha-fetoprotein level C. Refer for liver transplantation D. Start glucocorticoids E. Start ursodeoxycholic acid

E. Start ursodeoxycholic acid - This is PBC - Ursodeoxycholic acid prolongs survival and should be initiated immediately upon recognition of disease

70 year is brought to the ED due to 3 hours of intense, constant chest and neck pain that radiates to the interscapular area. Sharp pain. 20 year history of hypertension and 10 year history of 2 diabetes mellitus. Appears anxious and uncomfortable. BP high in both arms. Examination shows an early decresendo diastolic murmur best heard at the left sternal border at the fourth intercostal space. ECG shows sinus tachycardia, voltage criteria for left ventricular hypertrophy and T wave inversions in leads V5 and V6. Serum creatinine is 2.1. Which of the following is the best next step in management of this patient? A. Coronary angiography B. Low dose aspirin and trop levels C. MR angiography D. Serum BNP E. TEE

E. TEE - You can use this or MR angiography - BUT he has renal insufficiency so you can't use contrast

Patient with symptoms concerning for IBS-C. What next? A. Celiac serology testing B. colonscopy C. CT scan of the abdomen and pelvis D. fecal calprotectin test E. Trial of fiber supplementation F. Tricyclic antidepressant

E. Trial of fiber supplementation - Clinical diagnosis - Only screen for celiac if they're having diarrhea - Imaging only if concerned for malignancy, diverticulitis, or chronic pancreatitis - Fecal calprotectin - used to identify inflammatory causes of diarrhea - if patient has fever, leukocytosis, bleeding, or nocturnal diarrhea - You can use TCAs to treat IBS, but reserved for if if other treatments fail because of the side effects

45 year old man recently immigrated to the US from China. Dyspnea, fatigue, abdominal distension for 2 months. No other medical problems. Has worked as a farmer his whole life. Vital signs normal. He has pedal edema, increased abdominal girth, elevated JVP without inspiratory decline. He has decreased heart sounds and an accentuated sound directly after the second heart sound in early diastole. CXR demonstrates a calcification around the heart and JVP tracings show prominent x and y descents. WHat is the most likely cause? A. Cor pulmonale B. Pneumoconiosis C. Psittacosis D. Trpanosoma cruzi infection E. Tuberculosis

E. Tuberculosis - Calcification around heart + signs of venous overload + sharp x and y descents = constrictive pericarditis - In developing countries, tuberculosis is a classica cause of constrictive pericarditis

67 year old woman. progressive fatigue and anorexia over the last 6 months. lost 8.8 lbs, says she has early satiety. no abdominal pain, night sweats or fevers. has hypothyroidism, hypertension and gout. No tobacco/drug/alcohol use. Temp is 99.2, blood pressure and pulse are normal. No lymphadenopathy. A spleen tip is palpable with deep exhalation. Lab results show low hemoglobin, normal platelets, and high leukocytes (42,800). FISH reveals an abnormaltiy in chromsome 22. Which of the following is the most important target of this patient's disease? A. DNA methylation mutations B. Folic acid metabolism C. Retinoic acid receptor D. Thymidine synthesis E. tyrosine kinase

E. Tyrosine kinase - This is CML 9:22 translocation - BCR-ABL fusion protein - causes increased activity of tyrosine kinase

45 year old woman severe abdominal pain, nausea and 2 episodes of vomiting over the last 2 hours. Patient has had episodic epigastric pain and RUQ pain associated with nausea . Takes NSAIDs for headaches. Temp is 100, BP 140/92, pulse 102. Diffuse abdomianl tenderness and guarding. Stool guaiac positive. Next best step in management? A. Gallbladder ultrasound B. Mesenteric angiography C. NSAID avoidance and oral PPIs D. Upper and lower GI endoscopy E. upright x-ray of the chest and abdomen

E. Upright xray of the chest and abdomen - Peptic ulcer disease complicated by perforation - Results in chemical peritonitis (marked abdominal tenderness with guarding), fever, tachycardia

32 year old man. follow-up elevated blood pressure. Week ago, seen in the ED for nasal bleeding and found to be hypertensive. No PMH. BP is 150/95, repeat BP measurements demonstrate similar readings. Pulse 96 and regular. BMI 27. No heart murmurs. A fine hand tremor is present. No skin rashes. Lab results show normal TSH, normal serum electrolytes, and creatinine of 0.7. Which of the following is the best next step in evaluating this patient? A. duplex U/S of renal arteries B. In laboratory polysomnography C. Plasma fractioned metanephrines D. serum aldosterone level E. urine drug screen

E. Urine drug screen - Most likely surreptitious coccaine use - New-onset hypertension + epistaxis + hand tremor

A 31 year old woman comes to the office after 3 months of continuous burning upper abdominal pain that is only partially relieved by over-the-counter ranitidine or antacids. She also has experienced recent-onset constipation and excess urination. The patient reports no weight loss or decreased appetite. Her menstrual periods are regular. The patient's father has a history of "many stomach ulcers and multiple kidney stones." Her temperature is 98.9, blood pressure 130/85, pulse 78, respirations are 14. Abdominal examination shows normal bowel sounds, tenderness in the epigastric region, and no palpable massess. Her stools are positive for occult blood. Lab results shows hypercalcemia and low phosphate. Additional evaluation is most likely to reveal which of the following? A. Adrenal insufficiency B. Glucagonoma C. Medullary thyroid cancer D. Metastatic gastric carcinoma E. Milk alkali syndrome F. Primary hyperparathyroidism G. Sarcoidosis H. Thyroid adenoma

F. Primary hyperparathyroidism - This is MEN1 syndrome - Pituitary tumor + primary hyperparathyroidism + pancreatic/gastrointestinal neuroendocrine tumors - Parathyroid adenoma can cause constipation, polyuria and abdominal pain from hypercalcemia caused by the hyperparathyroidism - Burning upper abdominal pain only partially relieved by those drugs is concerning for Zollinger-Ellison syndrome

Patient has symptoms of atrial fibrillation and hyperthryoidism. ECG shows atrial fibrillation. What is the best medication to use? A. Adenosine B. Amiodarone C. Digoxin D. Flecainide E. Ibutilide F. Propranolol G. Synchornized cardioversion H. Verapamil

F. Propranolol - Even though you can use beta blockers or calcium channel blockers for rate control in A. fib, you want to use a beta blocker in this case because of the hyperthyroidism - Propranolol decreases conversion of T4 to T3 in peripheral tissues

47 year old male. mid-sternal chest pain and diaphoresis during a meeting. while waiting for EMS, compalins of dizziness and becomes unresponsive. coworkers perform CPR, ROSC in 60 seconds. Long history of T2DM, hypertension and hyperliipidemia. In the ED, ECG shows normal sinus rhythm, ventricular premature complexes, and a 3-mm ST segment elevation in leads V1-V3. What is the most likely primary path mechanisms responsible for this patient's syncopal episode? A. Asystole B. Atrial fibrillation C. Atrio-ventricular conduction block D. Paroxysmal supraventricular tachycardia E. Pulseless electrical activity F. Reentrant ventricular arrhythmias

F. Reentrant ventricular arrhythmias - Sudden cariac arrest in the setting of acute MI - ventricular arrhythmias are the most comon cause - Ventricular fibrillation is the most frequent one

68 year old man. routine visit. history of poorly controlled T2Dm and hypertension. Blood work 2 months ago showed hyperkalemia and lisinopril was discontinued. Current medications include glipizide, fuosemide, nifedipine and aspirin. Blood pressure 150/90, pulse 78. Repeat blood tests show sodium 136, potassium 5.8m, chloride 108, bicarbonate 18, BUN 28, creatinine 1.4. What is the most likely cause of this patients' electrolyte abnormalities? A. Furosemide use B. Laxative abuse C. Pre-renal azotemia D. Primary hyperaldosternoism E. REnal artery stenosis F. Renal tubular acidosis G. Surreptitious vomiting

F. Renal tubular acidosis - He has a non-anion gap acidosis - Non-anion gap MA + hyperkalemia that occur out of proportion to the renal dysfunction indicates renal tubular disorder

A 53 year old woman is brought to the ED after she collapsed suddnely while standing and lost consciousness for approximately 3 minutes. She recovered spontaneously but was very weak and dyspneic. She also reports left-sided chest pain. There was no tonic-clonic activity or incontinence. The patient was recently diagnosed wtih colon cancer on a routine colonscopy and is scheduled to undergo surcial resection. Her only other medical problem is hypertension but she has not been compliant with her medications. Her blood pressure is 86/50 and pulse is 120 and regular. Pulse oximetry is 80% on room air. She is diaphoretic and tachypeneic. Jugular venous pressure is 13. Lungs are clear. Further workup in this patient would most likely show what? A. Acute mitral regurgitation B. Complete herat block C. Left bundle branch block D. Mediastinal widening E. Pericardial tamponade F. Right ventricular dysfunction G. Tension pneumothorax

F. Right ventricular dysfunction - This is a massive PE - Look at her pulse ox and her sudden collapse - she also has a malignancy so she is predisposed to coagulation

55 year old woman. 3 weeks of fatigue and exertional SOB. Rheuamtic fever as a child. Progressive dental caries requiring several tooth extracitons; last one 4 weeks ago. No drug use. Temp 100. Holosystolic murmur. Linear hemorrahge present under the right index nail. Most likely organism? A. Group B strep B. Enterococci C. Staph auerus D. Staph epidermidis E. Strep gallolyticus F. Strep mutans G. Strep pneumo

F. Strep mutans - Oral manipulation - think viridans strep - this is just a type of it

80 year old woman. progressive lower extrmeity edema over the last 2 months. Has become somewhat tired and short of breath with daily activities. stomach feels full and her appetite has been decreased. Blood pressure and pulse are normal. Jugular veins are distended with the patient in the seated position. No heart murmurs. Breath sounds decreased at the bases b/l. Abdomen is distended with flank dullness on percussion. 3+ peripheral edema. Labs show low hemoglobin and low sodium. Urine albumin excretion 1g/24 hr. Echo reveals left atrial enlargement, marked concentric left ventricular hypertrophy, and left ventricular ejection fraction of 70%. What is the most likely cause of her symptoms? A. hypertrophic obstructive cardiomyopathy B. inferior vena cava thrombosis C. ischemic cardiomyopathy D. protein malnutrition E. renal disease F. Restrictive cardiomyopathy G. Thiamine deficiency

F. restrictive cardiomyopathy - This is amyloidosis - proteinuria, anemia, nephrotic syndrome, early satiety, peripheral or autonomic neuropathy, enlarged tongue..

29 year old woman. 6 month history of numbness and pain in upper arms. worse on left side. fatigue, fleeting joint pains and 11 lb weight loss. emigrated from vietnam 5 years ago. BP 140/90 right arm, 90/55 left arm, pulse is 78. Conjunctival and oral mucosa are moist and without lesions. No heart murmurs. Bruit is heard in the right supraclavicular fossa and the left radial and brachial pulses are decreased. No skin lesions. ESR 40. Most likely diagnosis? A. Aortic dissection B. coarctation of the aorta C. giant cell arteritis D. kawasaki disease E. polymyositis F. takayasu arteritis G. thromboangiitis obliterans

F. takayasu arteritis - constituional + arterio-occlusive + arthralgias/myalgias - Blood pressure discrepancies, pulse deficitis, arterial bruits - Elevated inflammatory markers - Tx: systemic steroids

43 year old man. frequent epigastric burning not relieved by antacids for the past 4 months. Typically brought on by heavy lifting at work and takes 10-15 minutes to go away. No neck or arm pain, cough or SOB. Has SLE treated wtih prednisone and hydroxychloroquine. Lifetime nonsmoker. BP 140/90, herat 80 and regular. Breath soudns equal on both sides. No wheezes or crackles heard. S1 and S2 are present. No murmurs or rubs. ECG normal. What next? A. Abdominal CT B. Abdominal ultrasound C. Chest CT without contrast D. Coronary angiography E. Echo F. Ecophageal motility studies G. Exercise ECG H. Stool H. pylori antigen testing I. Upper EGD

G. Exercise ECG - This is atypical angina - angina without the typical chest discomfort - SLE is a known risk factor for accelearted atheroscelrosis and premature coronary artery disease - Use exercise ECG as initial stress test

58 year old man, hasn't seen a physicain in 10 years, comes to the office to establish care. Reports dry itchy skin over his legs and increased leg discomfort at the end of teh day. Has gained 20 lb of weight over the last 5 years. No PMH, takes no medications. Has smoked a pack of cigarettes daily for 30 years. BP 150/84, other vitals are normal. BMI is 34. The lower extremities are warm to the touch below the knees bilaterally and have weakly palpable peripheral pulses. Which of the following is most likely to identify the cause of this patient' skin condition? a. Ankle brachial index B. Hemoglobin A1c C. Inguinal lymph node biopsy D. serum TSH level E. Skin allergy testing F. Skin biopsy G. Venous doppler ultrasound

G. Venous doppler ultrasound - Chronic venous stasis - Ultrasound not routinely performed, but can rule out venous thrombosis in patients with acute unilateral symptoms

What is leukoplakia?

White granular patch or plaque over the buccal mucosa in a patient with history of alcohol and tobacco use - Precancerous lesion - hyperplasia of the squamous epithelium


Conjuntos de estudio relacionados

anthropology 2019 concept questions

View Set

CSI Practice Guide Glossary of Terms #2

View Set

Chapter 7 B Law (Negligence and Strict Liability)

View Set

Introduction to Central Nervous System Pharmacology

View Set

ch 22 NCLEX questions, Chapter 22, Chapter 22: Neurodevelopmental Disorders, Psych CH 22

View Set

Texas Principles of Real Estate 2 - Chp. 2 Real Estate Appraisal

View Set

BJU Earth Science Fourth Edition Chapter 22 Review

View Set

Queen Elizabeth 1 - interesting and useful facts

View Set

Ch.15 Homework Personal Safety and Injury Prevention

View Set

Intro to Computers - Ch 13 (book)

View Set